OB Final exam Nclex

Lakukan tugas rumah & ujian kamu dengan baik sekarang menggunakan Quizwiz!

The capacity of the uterus in a term pregnancy is how many times its prepregnant capacity? Record your answer as a whole number. ______ times

ANS: 500 The prepregnant capacity of the uterus is about 10 mL, and it reaches 5000 mL (5 L) by the end of the pregnancy, which reflects a 500-fold increase.

Calculate the estimated date of birth (EDD) in October using Nägele's rule for a patient whose last normal menstrual period (LNMP) began on January 1. Record your answer as a whole number. _______

ANS: 8 Nägele's rule is often used to establish the EDD. This method involves subtracting 3 months from the date that the LNMP began, adding 7 days, and then correcting the year, if appropriate. Subtracting 3 months from January 1 gives you the month of October and adding 7 days = October 8

In a prenatal education class, the nurse is reviewing the importance of using relaxation techniques during labor. Which patient statement will the nurse need to correct? a. "We will practice relaxation techniques only in a quiet setting so I can focus." b. "Relaxation is important during labor because it will help me conserve my energy." c. "If I relax in between contractions, my baby will get more oxygen during labor." d. "My partner and I will practice relaxation throughout the remainder of my pregnancy."

ANS: A Relaxation exercises must be practiced frequently to be useful during labor. Couples begin practice sessions in a quiet, comfortable setting. Later, they practice in other places that simulate the noise and unfamiliar setting of the hospital. The ability to relax during labor is an important component of coping effectively with childbirth. Relaxation conserves energy, decreases oxygen use, and enhances other pain relief techniques. Women learn exercises to help them recognize and release tension. The labor partner assists the woman by providing feedback during exercise sessions and labor.

Which laboratory result would be a cause for concern if exhibited by a patient at her first prenatal visit during the second month of her pregnancy? a. Rubella titer, 1:6 b. Platelets, 300,000/mm3 c. White blood cell count, 6000/mm3 d. Hematocrit 38%, hemoglobin 13 g/dL

ANS: A A rubella titer of less than 1:8 indicates a lack of immunity to rubella, a viral infection that has the potential to cause teratogenic effects on fetal development. Arrangements should be made to administer the rubella vaccine after birth during the postpartum period because administration of rubella, a live vaccine, would be contraindicated during pregnancy. Women receiving the vaccine during the postpartum period should be cautioned to avoid pregnancy for 3 months. The lab values for WBCs, platelets, and hematocrit/hemoglobin are within the expected range for pregnant women

Which complaint made by a patient at 35 weeks of gestation requires additional assessment? a. Abdominal pain b. Ankle edema in the afternoon c. Backache with prolonged standing d. Shortness of breath when climbing stairs

ANS: A Abdominal pain at 35 weeks gestation may indicate preeclampsia, or abruptio placentae. Ankle edema in the afternoon is a normal finding at this stage of the pregnancy. Backaches while standing is a normal finding in the later stages of pregnancy. Shortness of breath is an expected finding at 35 weeks.

In teaching a pregnant adolescent about nutrition, what should the nurse include in the care plan? a. Determine the weight gain needed to meet adolescent growth and add 35 lb. b. Suggest that she does not eat at fast food restaurants to avoid foods of poor nutritional value. c. Realize that most adolescents are unwilling to make dietary changes during pregnancy. d. Emphasize the need to eliminate common teen snack foods because they are too high in fat and sodium.

ANS: A Adolescents should gain in the upper range of the recommended weight gain. They also need to gain weight that would be expected for their own normal growth. Adolescents are willing to make changes; however, they still need to be like their peers. Eliminating fast foods will make her appear different from her peers. She should be taught to choose foods that add needed nutrients. Changes in the diet should be kept at a minimum and snacks should be included. Snack foods can be included in moderation and other foods added to make up for the lost nutrients.

A 36-year-old divorcee with a successful modeling career finds out that her 18-year-old daughter is expecting her first child. Which is a major factor in determining how this woman will respond to becoming a grandmother? a. Her age b. Her career c. Being divorced d. Age of the daughter

ANS: A Age is a major factor in determining the emotional response of prospective grandparents. Young grandparents may not be happy with the stereotype of grandparents as being old. Career responsibilities may have demands that make the grandparents not as accessible but are not a major factor in determining the woman's response to becoming a grandmother. Being divorced is not a major factor that determines the adaptation of grandparents. The age of the daughter is not a major factor that determines the adaptation of grandparents. The age of the grandparent is a major factor

A patient postdelivery is concerned about getting back to her prepregnancy weight as soon as possible. She had only gained 15 lb during her pregnancy. Which assessment factor would be of concern at her 6-week postpartum checkup? a. Patient has lost 30 lb during the 6-week period prior to her scheduled checkup. b. Patient states that she is eating healthy and limiting intake of processed foods. c. Patient relates increased consumption of fruits and vegetables in her diet postbirth. d. Patient has resumed her usual exercise pattern of walking around the neighborhood for 10 minutes each night.

ANS: A Although a certain amount of weight loss is expected in the postpartum period, the fact that the reported weight loss is double the amount of weight gained during the pregnancy places the patient at risk for malnutrition. Further inquiry is needed. Limiting the intake of processed foods is a healthy dietary alternative to decreasing sodium intake. Increases in fruits and vegetables are a healthy dietary alternative to decrease possible occurrence of hypertension. An exercise program is part of a healthy nutrition approach.

A pregnant patient would like to know which foods, other than dairy products, contain the most calcium. Which food group would the nurse recommend? a. Legumes b. Lean meat c. Whole grains d. Yellow vegetables

ANS: A Although dairy products contain the greatest amount of calcium, it can also be found in legumes, nuts, dried fruits, and some dark green leafy vegetables. Lean meats are rich in protein and phosphorus. Whole grains are rich in zinc and magnesium. Yellow vegetables are rich in vitamin A.

An expectant mother, diagnosed with oligohydramnios, asks the nurse what this condition means for the baby. Which statement should the nurse provide for the patient? a. Oligohydramnios can cause poor fetal lung development. b. Oligohydramnios means that the fetus is excreting excessive urine. c. Oligohydramnios could mean that the fetus has a gastrointestinal blockage. d. Oligohydramnios is associated with fetal central nervous system abnormalities

ANS: A Because an abnormally small amount of amniotic fluid restricts normal lung development, the fetus may have poor fetal lung development. Oligohydramnios may be caused by a decrease in urine secretion. Excessive amniotic fluid production may occur when the gastrointestinal tract prevents normal ingestion of amniotic fluid. Excessive amniotic fluid production may occur when the fetus has a central nervous system abnormality.

Which effect is a common response to both smoking and cocaine use in the pregnant patient? a. Vasoconstriction b. Increased appetite c. Increased metabolism d. Changes in insulin metabolism

ANS: A Both smoking and cocaine use cause vasoconstriction, which results in impaired placental blood flow to the fetus. Smoking and cocaine use do not increase appetite, change insulin metabolism, or increase metabolism.

Which information should the nurse stress in teaching a patient how best to relieve the symptoms of premenstrual syndrome (PMS)? a. Decrease her consumption of caffeine. b. Drink a small glass of wine with her evening meal. c. Decrease her fluid intake to prevent fluid retention. d. Eat three large meals a day to maintain glucose levels.

ANS: A Caffeine increases irritability, insomnia, anxiety, and nervousness. Alcohol aggravates depression and should be avoided in the patient with PMS. The patient should drink at least 2000 mL of water p

A pregnant patient has lactose intolerance. Which recommendation will the nurse provide to best help the patient meet dietary needs for calcium? a. Add foods such as nuts, dried fruit, and broccoli to the diet. b. Consume dairy products but take an over-the-counter anti-gas product. c. Increase the intake of dark leafy vegetables, such as spinach and chard. d. Use powdered milk instead of liquid forms of milk.

ANS: A Calcium is present in legumes, nuts, dried fruits, and broccoli, so these foods can be added to increase calcium intake. Although dark leafy vegetables contain calcium, they also contain oxalates that decrease the availability of calcium. Powdered milk contains lactase, similar to the nondehydrated varieties. Milk products should be avoided by patients with lactose intolerance. Adequate calcium may be obtained from food and supplements. Some patients may be able to tolerate lactose free dairy products.

The most appropriate statement for introducing the topic of family planning in the postpartum setting is a. "What are your plans for future pregnancies?" b. "Do you plan on being sexually active in the future?" c. "Let's talk about birth control, because breastfeeding is not 100% effective for preventing pregnancy." d. "Here are some pamphlets on available methods of birth control. I'll come back later and discuss them with you."

ANS: A Discussing future pregnancy plans opens the conversation to ways of preventing pregnancy from occurring before the woman is ready to have another child. "Do you plan on being sexually active in the future?" will only provide a yes or no answer and not allow for conversation. The family needs to be ready to talk about birth control; the effect of breastfeeding on birth control is applicable only to the woman. Pamphlets are not always the best form of teaching. The patient is usually too tired and overwhelmed to read more information in the immediate postpartum period

Which statement best describes the changes that occur during the fetal period of development? a. Maturation of organ systems b. Development of basic organ systems c. Resistance of organs to damage from external agents d. Development of placental oxygen-carbon dioxide exchange

ANS: A During the fetal period, the body systems grow in size and mature in function to allow independent existence after birth. Basic organ systems are developed during the embryonic period. The organs are always at risk for damage from external sources; however, the older the fetus, the more resistant will be the organs. The greatest risk is when the organs are developing. The placental system is complete by week 12, but that is not the best description of the fetal period.

What is the purpose of the ovum's zona pellucida? a. Prevents multiple sperm from fertilizing the ovum b. Stimulates the ovum to begin mitotic cell division c. Allows the 46 chromosomes from each gamete to merge d. Makes a pathway for more than one sperm to reach the ovum

ANS: A Fertilization causes the zona pellucida to change its chemical composition so that multiple sperm cannot fertilize the ovum. Mitotic cell division begins when the nuclei of the sperm and ovum unite. Each gamete (sperm and ovum) has only 23 chromosomes; there will be 46 chromosomes when they merge. Once sperm has entered the ovum, the zona pellucida changes to prevent other sperm from entering.

The labor nurse is reviewing breathing techniques with a primiparous patient admitted for induction of labor. When is the best time to encourage the laboring patient to use slow, deep chest breathing with contractions? a. During labor, when she can no longer talk through contractions b. During the first stage of labor, when the contractions are 3 to 4 minutes apart c. Between contractions, during the transitional phase of the first stage of labor d. Between her efforts to push to facilitate relaxation between contraction

ANS: A Focused breathing techniques should not be used in labor until they are actually needed, which is usually when the woman can no longer walk and talk during a contraction. If breathing techniques are used too early, the woman tends to move through the different techniques too quickly, and she may stop using them. In addition, the use of the more complex breathing patterns in latent labor may increase fatigue. Women should be encouraged to adapt the techniques to their own comfort and needs. Breathing deeply between contractions or pushing can increase the possibility of carbon dioxide retention and make the patient dizzy.

The procedure in which ova are removed by laparoscopy, mixed with sperm, and the embryo(s) returned to the woman's uterus is a. in vitro fertilization (IVF). b. tubal embryo transfer (TET). c. therapeutic insemination (IUI). d. gamete intrafallopian transfer (GIFT).

ANS: A In vitro fertilization is a procedure used to bypass blocked or absent fallopian tubes. Tubal embryo transfer places the conceptus into the fallopian tube. Therapeutic insemination, also known as intrauterine insemination, uses the partner's sperm or that of a donor and places it directly into the uterus. Gamete intrafallopian transfer is when the sperm and ova are placed in the fallopian tube

Which specific instruction should the nurse teach to assist a patient to regain control of her urinary sphincter? a. Perform Kegel exercises. b. Void every hour while awake. c. Drink 8 to 10 glasses of water each day. d. Allow the bladder to become distended before voiding

ANS: A Kegel exercises, tightening and relaxing the pubococcygeal muscle, will improve control of the urinary sphincter. A prescribed schedule may help; however, every hour is too frequent. Restricting fluids will cause bladder irritation, which exacerbates the problem. Drinking adequate fluids will decrease the concentration of urine; however, this intervention will not improve sphincter control. Overdistention of the bladder will result in further stress incontinence.

Which piece of the usual equipment setup for a pelvic examination is omitted with a Pap test? a. Lubricant b. Speculum c. Fixative agent d. Gloves and eye protectors

ANS: A Lubricants other than water or water-based lubricants on the cervix interfere with the accuracy of the cytology report. A speculum is necessary to visualize the cervix. A fixative agent is applied to the slide to prevent drying or disruption of the specimen. The examiner should always use standard precautions including gloves and eye protection.

A pregnant patient asks the nurse how her baby gets oxygen to breathe. What is the nurse's best response? a. "Oxygen-rich blood is delivered through the umbilical vein to the baby." b. "Take lots of deep breaths because the baby gets all of its oxygen from you." c. "You don't need to be concerned about your baby getting enough oxygen." d. "The baby's lungs are not mature enough to actually breathe, so don't worry."

ANS: A Oxygen-rich blood travels from the mother's circulatory system to the placenta and from the placenta to the umbilical vein (veins carry blood to the heart). From the vein, most of the oxygenated blood travels to the fetal liver or the inferior vena cava. Taking deep breaths can temporarily increase oxygenation but can also lead to increased carbon dioxide retention and dizziness. The patient is asking a normal fetal developmental question often asked by pregnant women. Fetal lungs reach maturity by 37 weeks of gestation, but fetal breathing movements are common. Oxygen transport across lung tissue occurs with the first breath.

Which guidance related to a healthy diet during pregnancy will the nurse provide to a patient in her 1st trimester? a. "Every day you need to have at least 6 ounces of protein from sources such as meat, fish, eggs, beans, nuts, soybean products, and tofu." b. "High-dose vitamin A supplements will promote optimal vision while preventing a common cause of blindness in neonates." c. "Meals such as sushi with a cold deli salad made with raw sprouts combine high-fiber foods with protein sources to meet multiple nutritional needs." d. "Vitamin and mineral supplements can meet your nutrient needs if you have inadequate intake because of nausea or a sensation of fullness."

ANS: A Protein sources include meat, poultry, fish, eggs, legumes (e.g., beans, peas, lentils), nuts, and soybean products such as tofu. Pregnant women need 6 to 6.5 oz of protein daily. Vitamin A can cause fetal anomalies of the bones, urinary tract, and central nervous system when taken in high doses. Pregnant women should avoid raw fish and foods such as cold deli salads and raw sprouts. Supplements do not generally contain protein and calories and may lack many necessary nutrients; therefore they cannot serve as food substitutes.

A couple who has not achieved a successful pregnancy is scheduled to meet with a fertility specialist. Which simple evaluation is usually the first test to be performed? a. Semen analysis b. Testicular biopsy c. Endometrial biopsy d. Hysterosalpingography

ANS: A Semen analysis is usually the first test to be performed because it is least costly and noninvasive. Endometrial biopsy determines whether the endometrium is responding to ovarian stimulation. A testicular biopsy is an invasive examination using a local anesthetic. Hysterosalpingography uses a contrast medium to evaluate the structure and patency of the uterus and tubes.

When a pregnant woman develops ptyalism, which guidance should the nurse provide? a. Chew gum or suck on lozenges between meals. b. Eat nutritious meals that provide adequate amounts of essential vitamins and minerals. c. Take short walks to stimulate circulation in the legs and elevate the legs periodically. d. Use pillows to support the abdomen and back during sleep.

ANS: A Some women experience ptyalism, or excessive salivation. The cause of ptyalism may be decreased swallowing associated with nausea or stimulation of the salivary glands by the ingestion of starch. Small frequent meals and use of chewing gum and oral lozenges offer limited relief for some women. All other options include recommendations for pregnant women; however, they do not address ptyalism.

The traditional diet of Asian women includes little meat and few dairy products and may be low in calcium and iron. The nurse can assist a patient increase her intake of these foods by which action? a. Suggest that she eat more tofu, bok choy, and broccoli. b. Suggest that she eat more hot foods during pregnancy. c. Emphasize the need for increased milk intake during pregnancy. d. Tell her husband that she must increase her intake of fruits and vegetables for the baby's sake.

ANS: A The diet should be improved by increasing foods acceptable to the woman. These foods are common in the Asian diet and are good sources of calcium and iron. Pregnancy is considered hot; therefore the woman would eat cold foods. Because milk products are not part of this woman's diet, it should be respected and other alternatives offered. Also, lactose intolerance is common. Fruits and vegetables are cold foods and should be included in the diet. In regard to the family dynamics, however, the husband does not dictate to the wife in this culture

The nurse is assessing a newborn immediately after birth. After assigning the first Apgar score of 9, the nurse notes two vessels in the umbilical cord. What is the nurse's next action? a. Assess for other abnormalities of the infant. b. Note the assessment finding in the infant's chart. c. Notify the health care provider of the assessment finding. d. Call for the neonatal resuscitation team to attend the infant immediately.

ANS: A The normal finding in the umbilical cord is two arteries and one vein. Two vessels may indicate other fetal anomalies. Notation of the finding is the appropriate next step when the finding is expected. The health care provider will need to be notified; however, the infant is the nurse's primary concern and must be assessed for abnormalities first. The initial Apgar score is 9, indicating no signs of distress or need of resuscitation.

A patient with a BMI of 32 has a positive pregnancy test. What is the maximum number of pounds that the nurse will advise the patient gain during the pregnancy? a. 20 lb b. 25 lb c. 28 lb d. 40 lb

ANS: A The weight gain for obese women is 5 to 9 kg (11 to 20 lb). A BMI of 30 or higher categorizes the patient as obese. The other options refer to minimal or maximal weight gain for patients in other BMI categories.

A patient is using Depo-Provera as her method of birth control. Which clinical finding warrants immediate intervention by the nurse? a. Mid-cycle bleeding b. Nausea c. Temperature of 37.8°C (100°F) d. Irregular periods

ANS: A When using Depo-Provera, the major side effect is irregular bleeding. The presence of mid-cycle bleeding warrants further investigation at this time. Nausea, fever, and irregular periods are not the result of Depo-Provera.

Which physical characteristics decrease as the fetus nears term? (Select all that apply.) a. Vernix caseosa b. Lanugo c. Port wine stain d. Brown fat e. Eyebrows or head hair

ANS: A, B Both vernix caseosa and lanugo decrease as the fetus reaches term. Port wine stain is a birthmark and, if present, will be exhibited at or shortly after birth. Brown fat in the fetus will be maintained in order to maintain core temperature. Eyebrows and head hair increase as the fetus nears term.

While interviewing a 48-year-old patient during her annual physical examination, the nurse learns that she has never had a mammogram. The American Cancer Society recommends annual mammography screening starting at age 40. Before the nurse encourages this patient to begin annual screening, it is important for her to understand the reasons why women avoid testing. These reasons include which of the following? (Select all that apply.) a. Fear of x-ray exposure b. Expense of the procedure c. Reluctance to hear bad news d. Having heard that the test is painful e. Belief that lack of family history makes this test unnecessary

ANS: A, B, C, D Fear of x-ray exposure, expense, reluctance to hear bad news, and fear of pain are reasons women avoid having a mammogram done. Although the test is expensive, it is usually covered by health insurance. Many communities offer low-cost or free screening to women without insurance. It is important to acknowledge that some discomfort occurs with screening. Scheduling the test immediately at the end of a period makes it less painful. The risk of radiation exposure is minimal to none. Nurses play a vital role in providing information and reassurance to help women overcome these fears. Even patients with no family history should have a regular screening done. The nurse should emphasize that a combination of breast self-examination and mammography needs to be performed at regular intervals. Women with a family history may need to begin screening at a younger age and have additional testing such as ultrasound performed

The nurse is teaching a breastfeeding patient about substances to avoid while she is breastfeeding. Which substances should the nurse include in the teaching session? (Select all that apply.) a. Caffeine b. Alcohol c. Omega-6 fatty acids d. Appetite suppressants e. Polyunsaturated omega-3 fatty acid

ANS: A, B, D Foods high in caffeine should be limited. Infants of mothers who drink more than two or three cups of caffeinated coffee or the equivalent each day may be irritable or have trouble sleeping. Although the relaxing effect of alcohol was once thought to be helpful to the nursing mother, the deleterious effects of alcohol are too important to consider this suggestion appropriate today. An occasional single glass of an alcoholic beverage may not be harmful, but larger amounts may interfere with the milk-ejection reflex and may be harmful to the infant. Nursing mothers should avoid appetite suppressants, which may pass into the milk and harm the infant. The long-chain polyunsaturated omega-3 and omega-6 fatty acids are present in human milk. Therefore they should be included in the mother's diet during lactation.

The nurse is teaching a pregnant patient about signs of possible pregnancy complications. Which should the nurse include in the teaching plan? (Select all that apply.) a. Report watery vaginal discharge. b. Report puffiness of the face or around the eyes. c. Report any bloody show when you go into labor. d. Report visual disturbances, such as spots before the eyes. e. Report any dependent edema that occurs at the end of the day.

ANS: A, B, D Watery vaginal discharge could mean that the membranes have ruptured. Puffiness of the face or around the eyes and visual disturbances may indicate preeclampsia or eclampsia. These three signs should be reported. Bloody show as labor starts may mean the mucus plug has been expelled. One of the earliest signs of labor may be bloody show, which consists of the mucus plug and a small amount of blood. This is a normal occurrence. Up to 70% of women have dependent edema during pregnancy. This is not a sign of a pregnancy complication.

Which findings are presumptive signs of pregnancy? (Select all that apply.) a. Quickening b. Amenorrhea c. Ballottement d. Goodell's sign e. Chadwick's sign

ANS: A, B, E Quickening, amenorrhea, and Chadwick's sign are presumptive signs of pregnancy. Ballottement and Goodell's sign are probable signs of pregnancy.

The nurse is teaching a pregnant patient about food safety during pregnancy and lactation. Which statements by the patient indicate she understood the teaching? (Select all that apply.) a. "I will limit my intake of shrimp to 12 oz a week." b. "I will avoid the soft cheeses made with unpasteurized milk." c. "I plan to continue to pack my bologna sandwich for lunch." d. "I am glad I can still go to the sushi bar during my pregnancy." e. "I will not eat any swordfish or shark while I am pregnant or nursing."

ANS: A, B, E Statements that indicate the patient understood the teaching are limiting shrimp to 12 oz a week, avoiding soft cheeses, and not eating any swordfish. A bologna sandwich should be avoided unless it is reheated until steaming hot. Raw or undercooked fish should be avoided.

The nurse is assessing a patient in her 37th week of pregnancy for the psychological responses commonly experienced as birth nears. Which psychological responses should the nurse expect to evaluate? (Select all that apply.) a. The patient is excited to see her baby. b. The patient has not started to prepare the nursery for the new baby. c. The patient expresses concern about how to know if labor has started. d. The patient and her spouse are concerned about getting to the birth center in time. e. The patient and her spouse have not discussed how they will share household tasks

ANS: A, C, D As birth nears, the expectant patient will express a desire to see the baby. Most pregnant patients are concerned with their ability to determine when they are in labor. Many couples are anxious about getting to the birth facility in time for the birth. As birth nears, a nesting behavior occurs, which means getting the nursery ready. Not preparing the nursery at this stage is not a response that the nurse should expect to assess. Negotiation of tasks is done during this stage. Discussion regarding the division of household chores is not a response that the nurse should expect to assess at this stage.

The nurse is advising a lactose-intolerant pregnant patient about calcium intake. Which calcium sources are approximately equivalent to 1 cup of milk? (Select all that apply.) a. 3/4 cup yogurt b. 1 cup of sherbet c. 1 1/4 oz of hard cheese d. 1 1/4 cups of ice cream e. 3/4 cup of low-fat cottage cheese

ANS: A, C, D Calcium sources approximately equivalent to 1 cup of milk include cup yogurt, ounce of hard cheese, and cups of ice cream. It takes 3 cups of sherbet and cups of low-fat cottage cheese to equal the calcium equivalent of 1 cup of milk.

A pregnant patient reports that she works in a long-term care setting and is concerned about the impending flu season. She asks about receiving the flu vaccine. As the nurse, you are aware that some immunizations are safe to administer during pregnancy, whereas others are not. Which vaccines could this patient receive? (Select all that apply.) a. Tetanus b. Varicella c. Influenza d. Hepatitis A and B e. Measles, mumps, rubella (MMR)

ANS: A, C, D Inactivated vaccines such as those for tetanus, hepatitis A, hepatitis B, and influenza are safe to administer to women who have a risk for contracting or developing the disease. Immunizations with live virus vaccines such as MMR, varicella (chickenpox), or smallpox are contraindicated during pregnancy because of the possible teratogenic effects on the fetus.

Healthy People 2020 goals directed at women's health issues focus on which areas? (Select all that apply.) a. Increased screening for cervical and colorectal cancers b. Reduction of cancer survivor rate based on clinical management treatment c. Decreased morbidity and mortality related to breast cancer d. Reduction in hospitalization for hip fractures in the older female population e. Reduction in deaths associated with cardiovascular causes such as stroke and coronary artery disease (CAD)

ANS: A, C, D, E Healthy People 2020 goals directed at women's health focus on increased access to screening for cervical and colorectal cancers, decreased deaths occurring from breast cancer and heart disease, and decreased hospitalization for hip fractures. A reduction of the cancer survival rate would reflect increased morbidity and mortality.

A 38-year-old patient presents to the clinic office complaining of increased bilateral tenderness of her breasts prior to the onset of menses. On questioning the patient, this presentation has occurred off and on for several years; however, the pain has increased. Physical examination reveals lumpy areas bilaterally on the upper outer quadrants of each breast tissue. The areas of concern are approximately 2 cm in size. Based on this assessment, which diagnostic testing would be necessary? (Select all that apply.) a. Ultrasound examination b. Open biopsy c. Fine-needle aspiration (FNA) biopsy d. CBC with differential e. Mammogram

ANS: A, C, E Based on the clinical presentation, the patient may have fibrocystic breast disease. Although this condition is typically benign, the fact that the patient has noted a change in tenderness should be evaluated. Ultrasound, FNA, and mammography may be indicated to provide a baseline for comparison and rule out any malignancy. An open or surgical biopsy is not indicated at the present time but may be needed if the other test results indicate any pathology. Blood work is not indicated at this time relative to the diagnosis.

The prenatal nurse educator is teaching couples the technique of applying sacral pressure during labor. Which should be included in the teaching session? (Select all that apply.) a. The technique can be combined with heat to the area. b. A jiggling motion should be used while applying the pressure. c. Tennis balls may be used to apply the pressure to the sacral area. d. The pressure against the sacrum should be intermittent during the contraction. e. The hand may be moved slowly or remain positioned directly over the sacrum

ANS: A, C, E Sacral pressure can be combined with thermal stimulation to increase effectiveness. The hand may be moved slowly over the area or remain positioned directly over the sacrum, but pressure should be continuous and firm throughout the contraction. Care should be taken not to jiggle the woman, which may be irritating

The nurse is planning care for a patient in her first trimester of pregnancy. The patient is experiencing nausea and vomiting. Which interventions should the nurse plan to share with this patient? (Select all that apply.) a. Suck on hard candy. b. Take prenatal vitamins in the morning. c. Try some herbal tea to relieve the nausea. d. Drink fluids frequently but separate from meals. e. Eat crackers or dry cereal before arising in the morning

ANS: A, D, E A patient experiencing nausea and vomiting should be taught to suck on hard candy, drink fluids frequently but separately from meals, and eat crackers, dry toast, or dry cereal before arising in the morning. Prenatal vitamins should be taken at bedtime because they may increase nausea if taken in the morning. Before taking herbal tea, the patient should check with her health care provider

Which adverse reactions are associated with the administration of clomiphene citrate (Clomid)? (Select all that apply.) a. Abdominal bloating b. Diarrhea c. Oliguria d. Nausea and vomiting e. Abnormal uterine bleeding

ANS: A, D, E Some adverse reactions associated with Clomid are abdominal distension, frequent urination, nausea and vomiting, and abnormal uterine bleeding. Diarrhea is not a common presentation.

A nurse is conducting prenatal education classes for a group of expectant parents. Which information should the nurse include in her discussion of the purpose of amniotic fluid? (Select all that apply.) a. Cushions the fetus b. Protects the skin of the fetus c. Provides nourishment for the fetus d. Allows for buoyancy for fetal movement e. Maintains a stable temperature for the fetus

ANS: A, D, E The amniotic fluid provides cushioning for the fetus against impacts to the maternal abdomen. It provides a stable temperature and allows room and buoyancy for fetal movement. Vernix caseosa, the cheeselike coating on the fetus, provides skin protection. The placenta provides nourishment for the fetus.

Which factors contribute to the presence of edema in the pregnant patient? (Select all that apply.) a. Diet consisting of processed foods b. Hemoconcentration c. Increase in colloid osmotic pressure d. Last trimester of pregnancy e. Decreased venous return

ANS: A, D, E Processed foods, which are high in sodium content, can contribute to edema formation. As the pregnancy progresses, because of the weight of the uterus, compression takes place, leading to decreased venous return and an increase in edema formation. A decrease in colloid osmotic pressure would contribute to edema formation and fluid shifting. Hemodilution would also lead to edema formation

A patient presents to the Women's Health Clinic for continuation of her contraceptive method. She has been using Depo-Provera (medroxyprogesterone acetate) for 24 months. In preparation for instituting a plan of care, the nurse would consider which option as a priority? a. Schedule the patient for follow-up baseline diagnostic testing to confirm that the patient is not pregnant. b. Obtain information for an alternate contraception method. c. Ask the patient for additional information related to her menstrual cycle. d. Inspect the skin for site selection of contraceptive method.

ANS: B According to WHO (World Health Organization) guidelines, women should not be on Depo-Provera for more than 2 years due to loss of bone density. Therefore, the nurse should include assessments for other types of contraception methods for this patient. Although it may prove to be important to rule out possible pregnancy, based on the provided information, discontinuation of this method is the priority intervention. Depo-Provera can cause menstrual irregularities, but this is not the priority intervention. The nurse can follow up on this issue later. Because the current method of contraception should no longer be used, this option is not necessary.

While performing a self-breast exam, the patient notes an area on the right breast that is nodular, with some associated tenderness. This is a new onset finding because the exams were not problematic in the past. The left breast examination is unremarkable. The patient calls to report her findings to the clinical nurse because this is not her typical result. What action should the nurse perform next? a. Refer the patient to an oncologist because the results are suspicious. b. Ask the patient to come in for an office visit so that the findings can be validated but tell her that this information is within the normal range of presentation. c. Have the patient wear a tight-fitting bra and tell her that the tenderness is associated with ovulation and will pass. d. Have the patient repeat the self-breast exam in 2 weeks and call back with findings to provide a basis for comparison.

ANS: B Although these findings are within the normal range of presentation for breast tissue, they are not in the normal presentation for this patient. The patient has called to express concern; therefore, the nurse should have the patient schedule an appointment for assessment and evaluation. There is no need for referral to a specialist at this time. Wearing a tight-fitting bra may help provide support; however, does not address the physical findings and concern of the patient. Repeating the self-breast exam may be required; however, it does not address the patient's current concerns.

Which physiologic finding is consistent with normal pregnancy? a. Systemic vascular resistance increases as blood pressure decreases. b. Cardiac output increases during pregnancy. c. Blood pressure remains consistent independent of position changes. d. Maternal vasoconstriction occurs in response to increased metabolism

ANS: B Cardiac output increases during pregnancy as a result of increased stroke volume and heart rate. Systemic vascular resistance decreases while blood pressure remains the same. Maternal blood pressure changes in response to patient positioning. In response to increased metabolism, maternal vasodilation is seen during pregnancy.

A pregnant patient arrives for her first prenatal visit at the clinic. She informs the nurse that she has been taking an additional 400 mcg of folic acid prior to becoming pregnant. Based on the patient's history, she has reached 8 weeks' gestation. Which recommendation would the nurse provide regarding folic acid supplementation? a. Have the patient continue to take 400 mcg folic acid throughout her pregnancy. b. Tell the patient that she no longer has to take additional folic acid because it will be included in her prenatal vitamins. c. Have the patient increase her folic acid intake to 1000 mcg throughout the rest of her pregnancy. d. Schedule the patient to go for an AFP (alpha-fetoprotein) test.

ANS: B Prenatal vitamins include adequate folic acid supplementation, so patients should not take additional supplementation as long as they continue their prenatal vitamins. During pregnancy, the recommendation is to increase the folic acid intake to 600 mcg. 1000 mcg of folic acid would be an excessive dose. The AFP test should be done at 15 to 18 weeks' gestation. This is not clinically indicated because the patient is at 8 weeks' gestation.

Which patient is most likely to develop osteoporosis? a. A 50-year-old patient on estrogen therapy b. A 55-year-old patient with a sedentary lifestyle c. A 65-year-old patient who walks 2 miles each day d. A 60-year-old patient who takes supplemental calcium

ANS: B Risk factors for the development of osteoporosis include smoking, alcohol consumption, sedentary lifestyle, family history of the disease, and a high-fat diet. A number of drug therapies are available to reduce the progression of osteoporosis. Weight-bearing exercises have been shown to increase bone density. Supplemental calcium will help prevent bone loss, especially when combined with vitamin D

When planning a healthy diet with a pregnant patient, what should the nurse's first action be? a. Teach the patient about MyPlate. b. Review the patient's current dietary intake. c. Instruct the patient to limit the intake of fatty foods. d. Caution the patient to avoid large doses of vitamins, especially those that are fat-solubl

ANS: B The first action should be to assess the patient's current dietary pattern and practices because instruction should be geared to what she already knows and does. Teaching the food guide MyPlate is important but not the first action when planning a diet with a pregnant patient. Limiting intake of fatty foods is important in a pregnant patient's diet; however, not the first action. Caution regarding about excessive fat-soluble vitamins is important; however, not the first action.

A patient has had a prior history of endometriosis and comes to the clinic asking about which method of birth control might be helpful to alleviate her symptoms. Which birth control method would provide the greatest benefit to this patient? a. Withdrawal method b. Oral contraceptives c. Depo-Provera d. Intrauterine device (IUD)

ANS: B A patient who has a history of endometriosis may gain additional benefit from using an oral contraceptive as her birth control method because hormone levels will be more uniformly regulated with this type of treatment. The withdrawal method and Depo-Provera will not provide any additional benefit relative to a history of endometriosis. An IUD may cause further irritation to the endometrial lining so it would not be a prudent choice

A woman who has a successful career and a busy lifestyle will most likely look for which type of contraceptive? a. Requires extensive education to use b. Is the easiest and most convenient to use c. Costs the least d. Is permanent

ANS: B A woman who has a busy lifestyle will probably have less time to devote to contraceptive use, therefore she requires something that is easy and convenient. Extra time for education would not be acceptable for this woman. Cost would probably not be a problem. There is no indication that a woman who has a successful career and a busy lifestyle wants contraception to be permanent, simply convenient

A pregnant woman of normal weight enters her 13th week of pregnancy. If the patient eats and exercises as directed, what will the nurse anticipate as the ongoing weight gain for the remaining trimesters? a. 0.3 lb every week b. 1 lb every week c. 1.8 lb every week d. 2 lb every week

ANS: B After the first 12 weeks (first trimester), the pregnant woman should gain 0.35 to 0.5 kg (0.8 to 1 lb) per week for the remainder of the pregnancy

You are performing assessments for an obstetric patient who is 5 months pregnant with her third child. Which finding would cause you to suspect that the patient was at risk? a. Patient states that she doesn't feel any Braxton Hicks contractions like she had in her prior pregnancies. b. Fundal height is below the umbilicus. c. Cervical changes, such as Goodell's sign and Chadwick's sign, are present. d. She has increased vaginal secretions.

ANS: B Based on gestational age (20 weeks), the fundal height should be at the umbilicus. This finding is abnormal and warrants further investigation about potential risk. With subsequent pregnancies, multiparas may not perceive Braxton Hicks contractions as being evident compared with their initial pregnancy. Cervical changes such as Goodell's and Chadwick's signs should be present and are considered a normal finding. Increased vaginal secretions are normal during pregnancy as a result of increased vascularity.

. An expectant patient in her third trimester reports that she developed a strong tie to her baby from the beginning and now is really in tune to her baby's temperament. The nurse interprets this as the development of which maternal task of pregnancy? a. Learning to give of herself b. Developing attachment with the baby c. Securing acceptance of the baby by others d. Seeking safe passage for herself and her baby

ANS: B Developing a strong tie in the first trimester and progressing to be in tune is the process of commitment, attachment, and interconnection with the infant. This stage begins in the first trimester and continues throughout the neonatal period. Learning to give of herself is the task that occurs during pregnancy as the woman allows her body to give space to the fetus. She continues with giving to others in the form of food and presents. Securing acceptance of the baby is a process that continues throughout pregnancy as the woman reworks relationships. Seeking safe passage is the task that ends with birth. During this task, the woman seeks health care and carries out cultural practices.

When instructing a patient in the use of spermicidal foam or gel, it is important to include the information that a. effectiveness is about 85%. b. douching should be avoided for at least 6 hours. c. it should be inserted 1 to 2 hours before intercourse. d. one application is effective for several hours

ANS: B Douching within 6 hours of intercourse would remove the spermicide and increase the risk of pregnancy. Effectiveness of spermicidal foam or gel is only 72% when used alone. The spermicidal foam or gel should be inserted 15 minutes before intercourse. One application is effective for approximately 1 hour.

Which type of cutaneous stimulation involves massage of the abdomen? a. Imagery b. Effleurage c. Mental stimulation d. Thermal stimulation

ANS: B Effleurage is massage usually performed on the abdomen during contractions. Imagery exercises enhance relaxation by teaching the woman to imagine herself in a relaxing setting. Mental stimulation is a group of methods to decrease pain by increasing mental stimulation. Thermal stimulation decreases pain by using applications of heat and cold.

Which advice to the patient is one of the most effective methods for preventing venous stasis? a. Sit with the legs crossed. b. Rest often with the feet elevated. c. Sleep with the foot of the bed elevated. d. Wear elastic stockings in the afternoon.

ANS: B Elevating the feet and legs improves venous return and prevents venous stasis. Sitting with the legs crossed will decrease circulation in the legs and increase venous stasis. Elevating the legs at night may cause pressure on the diaphragm and increase breathing problems. Elastic stockings should be applied before lowering the legs in the morning.

A patient who is 7 months pregnant states, "I'm worried that something will happen to my baby." Which is the nurse's best response? a. "Your baby is doing fine." b. "Tell me about your concerns." c. "There is nothing to worry about." d. "The doctor is taking good care of you and your baby.

ANS: B Encouraging the patient to discuss her feelings is the best approach. The nurse should not disregard or belittle the patient's feelings. Responding that your baby is doing fine disregards the patient's feelings and treats them as unimportant. Responding that there is nothing to worry about does not answer the patient's concerns. Saying that the doctor is taking good care of you and your baby is belittling the patient's concerns.

The nurse is scheduling the next appointment for a healthy primigravida currently at 28 weeks gestation. When will the nurse schedule the next prenatal visit? a. 1 week b. 2 weeks c. 3 weeks d. 4 weeks

ANS: B From 29 to 36 weeks, routine prenatal assessment is every 2 weeks. If the pregnancy is high risk, the patient will see the health care provider more frequently.

A benign breast condition that includes dilation and inflammation of the collecting ducts is known as a. fibroadenoma. b. ductal ectasia. c. intraductal papilloma. d. chronic cystic disease

ANS: B Generally occurring in women approaching menopause, ductal ectasia results in a firm irregular mass in the breast, enlarged axillary nodes, and nipple discharge. Fibroadenoma is evidenced by fibrous and glandular tissues. They are felt as firm, rubbery, and freely mobile nodules. Intraductal papillomas develop in the epithelium of the ducts of the breasts; as the mass grows, it causes trauma or erosion within the ducts. Chronic cystic disease causes pain and tenderness. The cysts that form are multiple, smooth, and well delineated.

Which comment made by a new mother to her own mother is most likely to encourage the grandmother's participation in the infant's care? a. "Could you help me with the housework today?" b. "The baby is spitting up a lot. What should I do?" c. "I know you are busy, so I'll get John's mother to help me." d. "The baby has a stomachache. I'll call the nurse to find out what to do."

ANS: B Looking to the grandmother for advice encourages her to become involved in the care of the infant. Housework does not encourage the grandmother to participate in the infant's care. Getting John's mother to help and calling the nurse about advice excludes the grandmother.

A patient, age 49, confides in the nurse that she has started experiencing pain with intercourse. The patient asks, "Is there anything I can do about this?" The nurse's best response is a. "No, it is part of the aging process." b. "Water-soluble vaginal lubricants may provide relief." c. "You need to be evaluated for a sexually transmitted disease." d. "You may have vaginal scar tissue that is producing the discomfort."

ANS: B Loss of lubrication, with resulting discomfort in intercourse, is a symptom of estrogen deficiency. It is part of the aging process; however, the use of lubrication will help relieve the symptoms. This is a normal occurrence with the aging process and does not indicate an STD. It is caused by loss of lubrication with the decrease in estrogen. Scar tissue problems would have occurred earlier.

Which are the most common sites of breast cancer metastasis? a. Kidneys b. Bones and liver c. Heart and blood vessels d. Skin

ANS: B Metastasis occurs when the cancer cells spread by both blood and lymph system to distant organs and to vascular sites, commonly the lungs, liver, bones, and brain. Kidney metastasis is uncommon. Metastasis to the heart and blood vessels is uncommon. Skin cancer is not associated with breast cancer metastasis.

A patient in her first trimester complains of nausea and vomiting. The patient asks, "Why is this happening?" What is the nurse's best response? a. "It is due to an increase in gastric motility." b. "It may be due to changes in hormones." c. "It is related to an increase in glucose levels." d. "It is caused by a decrease in gastric secretions."

ANS: B Nausea and vomiting are believed to be caused by increased levels of hormones, decreased gastric motility, and hypoglycemia. Gastric motility decreases during pregnancy. Glucose levels decrease in the first trimester. Gastric secretions decrease, but this is not the main cause of nausea and vomiting.

Use Nägele's rule to determine the EDD (estimated day of birth) for a patient whose last menstrual period started on April 12. a. February 19 b. January 19 c. January 21 d. February 7

ANS: B Nägele's rule subtracts 3 months from the month of the last menstrual period (month 4 - 3 = January) and adds 7 days to the day that the last menstrual period started (April 12 + 7 days = April 19), so the correct answer is January 19 of the following calendar year.

Which symptom in a patient using oral contraceptives should be reported to the physician immediately? a. 5-lb weight gain b. Leg pain and edema c. Decrease in menstrual flow d. Increased pigmentation of the face

ANS: B Oral contraceptives increase clotting factors, which place the woman at risk for thrombophlebitis. Leg pain and edema are symptoms of thrombophlebitis. A 5-lb weight gain can be expected. A decrease in menstrual flow is an expected finding. Increased pigmentation of the face is a common finding.

An expectant mother says to the nurse, "When my sister's baby was born, it was covered in a cheese-like coating. What is the purpose of this coating?" The correct response by the nurse is to explain that the purpose of vernix caseosa is to a. regulate fetal temperature. b. protect the fetal skin from amniotic fluid. c. promote normal peripheral nervous system development. d. allow the transport of oxygen and nutrients across the amnion

ANS: B Prolonged exposure to amniotic fluid during the fetal period could result in breakdown of the skin without the protection of the vernix caseosa. The amniotic fluid aids in maintaining fetal temperature. Normal peripheral nervous system development is dependent on the nutritional intake of the mother. The amnion is the inner membrane that surrounds the fetus. It is not involved in the oxygen and nutrient exchange.

A patient who smokes one pack of cigarettes daily has a positive pregnancy test. The nurse will explain that smoking during pregnancy increases the risk of which condition? a. Congenital anomalies b. Death before or after birth c. Neonatal hypoglycemia d. Neonatal withdrawal syndrome

ANS: B Smoking during pregnancy increases the risk for spontaneous abortion, low birth weight, abruptio placentae, placenta previa, preterm birth, perinatal mortality, and SIDS. Smoking does not appear to cause congenital anomalies, hypoglycemia, or withdrawal syndrome.

A 45-year-old patient asks how often she should have a mammogram. The most appropriate answer is a. whenever she feels a lump. b. every year beginning at age 40. c. they are unnecessary until age 50. d. every year if you have risk factors.

ANS: B The American Cancer Society recommends that women have an annual mammogram after 40 years of age. Mammography should be done routinely following the American Cancer Society guidelines. Mammograms are necessary when a woman is in her 40s. Women with high-risk factors may require them more often

A patient in her fifth month of pregnancy asks the nurse, "How many more calories should I be eating daily?" What is the correct response by the nurse? a. 180 more calories a day b. 340 more calories a day c. 452 more calories a day d. 500 more calories a day

ANS: B The increased nutritional needs of pregnancy can be met with an additional 340 calories per day. 180 calories are not enough to meet the increased nutritional needs of pregnancy. 452 calories are more than the recommended calories for pregnancy at this gestation. A patient in her third trimester would increase her energy intake by 452 calories per day. 500 calories are more than the recommended calories for pregnancy.

Which of the following is a potential disadvantage for the patient who wishes to use an intrauterine device (IUD) as a method of birth control? a. Insertion of the device prior to coitus resulting in decreased spontaneity b. Ectopic pregnancy c. Protection against STDs d. Decrease in dysmenorrhea

ANS: B The insertion of an IUD is performed in a health care provider's office. An ectopic pregnancy can occur as a possible complication of the IUD. An IUD does not offer protection against STDs. A decrease in dysmenorrhea would be an advantage of using an IUD.

An expected change during pregnancy is a darkly pigmented vertical midabdominal line. The nurse recognizes this alteration as a. epulis. b. linea nigra. c. melasma. d. striae gravidarum.

ANS: B The linea nigra is a dark pigmented line from the fundus to the symphysis pubis. Epulis refers to gingival hypertrophy. Melasma is a different kind of dark pigmentation that occurs on the face. Striae gravidarum (stretch marks) are lines caused by lineal tears that occur in connective tissue during periods of rapid growth.

Which patient is most at risk for a low-birth-weight infant? a. 22-year-old, 60 inches tall, normal prepregnant weight b. 18-year-old, 64 inches tall, body mass index is <18.5 c. 30-year-old, 78 inches tall, prepregnant weight is 15 lb above the norm d. 35-year-old, 75 inches tall, total weight gain in previous pregnancies was 33 lb

ANS: B The patient who has a low prepregnancy weight is associated with preterm labor and low-birth-weight infants. Women who are underweight should gain more during pregnancy to meet the needs of pregnancy as well as their own need to gain weight; patients who have a normal prepregnancy weight, who start pregnancy overweight, or who have a history of excessive weight gain in pregnancy are not at risk for low-birth-weight infants.

A pregnant patient asks the nurse if she can double her prenatal vitamin dose because she does not like to eat vegetables. What is the nurse's response regarding the danger of taking excessive vitamins? a. Increases caloric intake b. Has toxic effects on the fetus c. Increases absorption of all vitamins d. Promotes development of pregnancy-induced hypertension (PIH)

ANS: B The use of vitamin supplements in addition to food may increase the intake of some nutrients to doses much higher than the recommended amounts. Overdoses of some vitamins have been linked to fetal defects. Vitamin supplements do not contain calories. Vitamin supplements do not have better absorption than natural vitamins and minerals. There is no relationship between vitamin supplements and PIH.

You are teaching a group of adolescents regarding myths and facts related to contraception. Which statement indicates that additional teaching is needed for this group? a. Adolescents are more likely to become pregnant even if they use available contraception methods correctly. b. The withdrawal technique provides a higher likelihood that a teen will not get pregnant. c. Pregnancy can occur in the presence or absence of orgasm. d. Pregnancy can occur even if a teen is menstruating at the time of coitus.

ANS: B The withdrawal technique does not decrease the likelihood of becoming pregnant for any woman including a teen. Withdrawal is a highly unreliable method of birth control. Even without penetration, ejaculation may result in pregnancy. Based on current clinical evidence, adolescents are more likely to become pregnant even if they correctly use available contraception methods. Pregnancy can occur in the presence or absence of orgasm, and also if the teen is menstruating at the time of coitus.

What is the term for the step in maternal role attainment that relates to the woman giving up certain aspects of her previous life? a. Fantasy b. Grief work c. Role playing d. Looking for a fit

ANS: B The woman experiences sadness as she realizes that she must give up certain aspects of her previous self and that she can never go back. This is called grief work. Fantasies allow the woman to try on a variety of possibilities or behaviors. This usually deals with how the child will look and the characteristics of the child. Role playing involves searching for opportunities to provide care for infants in the presence of another person. Looking for a fit is when the woman observes the behaviors of mothers and compares them with her own expectations.

Changes in the diet of the pregnant patient who has phenylketonuria would include a. adding foods high in vitamin C. b. eliminating drinks containing aspartame. c. restricting protein intake to <20 g a day. d. increasing caloric intake to at least 1800 cal/day.

ANS: B Use of aspartame by women with phenylketonuria can result in fetal brain damage because these women lack the enzyme to metabolize aspartame. Adding vitamin C, restricting protein, and increasing caloric intake are not necessary for the pregnant patient with phenylketonuria.

You are taking care of a patient who has had a colporrhaphy. Which option would indicate a priority assessment during the postoperative period? a. Documentation of a pessary in the operative procedure notes by the physician b. Removal of vaginal packing as ordered by the physician c. Use of a cell saver for transfusion therapy in the postoperative period d. Order for removal of staples 2 to 3 days post-procedure

ANS: B Vaginal packing is typically used in this type of pelvic surgery. The removal of the packing should be verified and documented. This is the priority assessment. A pessary would be utilized as a nonsurgical intervention for a patient who has had uterine prolapse and was not a surgical candidate based on medical history. A cell saver is used in orthopedic surgeries that are at risk for blood loss so that the patient's own blood can be re-infused based on established protocol. There are no staples used in this type of surgical procedure, which is also known as an A & P (anterior and posterior) repair

Which of these findings would indicate a potential complication related to renal function during pregnancy? a. Increase in glomerular filtration rate (GFR) b. Increase in serum creatinine level c. Decrease in blood urea nitrogen (BUN) d. Mild proteinuria

ANS: B With pregnancy, one would expect the serum creatinine and BUN levels to decrease. An elevation in the serum creatinine level should be investigated. With pregnancy, the GFR increases because of increased renal blood flow and is thus a normal expected finding. A decrease in the blood urea nitrogen level and mild proteinuria is expected findings in pregnancy.

Which of the following medical conditions could possible affect a woman's fertility status? a. Past medical history of asthma during childhood that is presently under control with the use of an inhaler b. Recently diagnosed with polycystic ovarian syndrome c. Past surgical history of removal of external polyps on labial tissue d. History of frequent sinus headaches that is seasonal in nature treated with over-the-counter medication

ANS: B Women with polycystic ovarian syndrome (PCOS) often have challenges conceiving in addition to other problems due to abnormal hormones and ovarian function. The use of inhaler therapy for the treatment of asthma should not affect the patient's fertility status. Removal of external polyps on the labia should not affect the patient's fertility. A history of sinus headaches should not affect the patient's fertility

The nurse is explaining fetal circulation to a group of nursing students. Which information should be included in the teaching session? (Select all that apply.) a. After birth the ductus venosus remains open, but the other shunts close. b. The foramen ovale shunts blood from the right atrium to the left atrium. c. The ductus venosus shunts blood from the liver to the inferior vena cava. d. The ductus arteriosus shunts blood from the right ventricle to the left ventricle.

ANS: B, C The foramen ovale shunts oxygenated blood from the right atrium to the left atrium, bypassing the lungs. The ductus venosus shunts oxygenated blood from the liver to the inferior vena cava. All shunts close after birth. The ductus arteriosus shunts blood from the right ventricle to the aorta.

The nurse is teaching a patient taking prenatal vitamins how to avoid constipation. Which should the nurse plan to include in the teaching session? (Select all that apply.) a. Advise taking a daily laxative for constipation. b. Recommend a diet high in fruits and vegetables. c. Encourage an increase in fluid consumption during the day. d. Increase the intake of whole grains and whole grain products. e. Suggest increasing the intake of dairy products, especially cheeses.

ANS: B, C, D Common sources of dietary fiber include fruits and vegetables (with skins when possible—apples, strawberries, pears, carrots, corn, potatoes with skins, and broccoli), whole grains, and whole grain products—whole wheat bread, bran muffins, bran cereals, oatmeal, brown rice, and whole wheat pasta. Increased intake of fluids can help prevent constipation. A pregnant patient should not take a daily laxative unless prescribed by her health care provider. Increased intake of dairy products, especially cheese, may increase constipation.

The nurse is planning a prenatal class on fetal development. Which characteristics of prenatal development should the nurse include for a fetus of 24 weeks, based on fertilization age? (Select all that apply.) a. Ear cartilage firm b. Skin wrinkled and red c. Testes descending toward the inguinal rings d. Surfactant production nears mature levels e. Fetal movement becoming progressively more noticeable

ANS: B, C, E A fetus of 24 weeks, based on fertilization age, will have wrinkled and red skin, testes descending toward inguinal rings, and the fetal movement becoming progressively more noticeable. Surfactant production nearing the mature levels does not occur until 32 weeks and ear cartilage is not firm until 38 weeks

Along with gas exchange and nutrient transfer, the placenta produces many hormones necessary for normal pregnancy, including which of the following? (Select all that apply.) a. Insulin b. Estrogen c. Progesterone d. Testosterone e. Human chorionic gonadotropin (hCG)

ANS: B, C, E HCG causes the corpus luteum to persist and produce the necessary estrogens and progesterone for the first 6 to 8 weeks. Estrogens cause enlargement of the woman's uterus and breasts and growth of the ductal system in the breasts and, as term approaches, plays a role in the initiation of labor. Progesterone causes the endometrium to change, providing early nourishment. Progesterone also protects against spontaneous abortion by suppressing maternal reactions to fetal antigens and reduces unnecessary uterine contractions. Other hormones produced by the placenta include hCT, hCA, and a number of growth factors. Insulin and testosterone are not secreted by the placenta

Which factors would contribute to abnormalities of the fallopian tube associated with the development of infertility? (Select all that apply.) a. History of conization of the cervix b. History of pelvic surgical procedures c. Incompetent cervix d. Past treatments of STD with follow-up test of cure e. Endometriosis

ANS: B, D, E Surgical procedures related to the cervix, along with an incompetent cervix, would not affect the fallopian tubes in terms of infertility. It would affect fertility issues related to the cervix as a result of potential scarring (conization) and an inability to maintain the pregnancy in the presence of an incompetent cervix. A history of pelvic surgical procedures could result in the development of pelvic adhesions, which would affect the fallopian tube. Also, the presence of STDs, even with effective treatment, along with the clinical diagnosis, would affect the fallopian tube and possibly result in infertility.

You are evaluating a patient in the clinic setting who has been taking oral contraceptives for several years, without side effects. Vital signs are stable and the patient denies any pain or tenderness. On examination, you note a small erythematous area of approximately 2 cm on her right lower leg. She denies any traumatic injury and says this is a recent onset of a few days. Based on this information you would a. instruct the patient to use warm compresses for several days and keep the leg elevated as much as possible. b. have the patient wear flats rather than heels to modify her gait and help alleviate this issue. c. refer the patient to the health care provider for additional diagnostic work up. d. have the patient take an over-the-counter (OTC) nonsteroidal antiinflammatory drug (NSAID) and return to the clinic if the problem persists.

ANS: C Because the patient has a history of oral contraceptive use, the nurse must assess and evaluate findings relative to ACHES (warning signs of oral contraceptives). Thus the patient should be worked up for the possibility of a superficial or deep vein thrombosis (DVT). Warm compresses and elevation of the leg, wearing flats, and taking an OTC NSAID may lead to further problems if there is an underlying clot that is not addressed promptly

Which situation best describes a man trying on fathering behaviors? a. Reading books on newborn care b. Spending more time with his siblings c. Coaching a little league baseball team d. Exhibiting physical symptoms related to pregnancy

ANS: C Coaching a little league baseball team shows interaction with children and assuming the behavior and role of a father. This best describes a man trying on the role of being a father. Men do not normally read information that is provided in advance. The nurse should be prepared to present information after the baby is born, when it is more relevant. The man will normally seek closer ties with his father. Exhibiting physical symptoms related to pregnancy is called couvade.

For the pregnant patient who is a vegan, what combination of foods will the nurse advise to meet the nutritional needs for all essential amino acids? a. Eggs and beans b. Fruits and vegetables c. Grains and legumes d. Vitamin and mineral supplements

ANS: C Combining incomplete plant proteins with other plant foods that have complementary amino acids allows intake of all essential amino acids. Dishes that contain grains (e.g., wheat, rice, corn) and legumes (e.g., garbanzo, navy, kidney, or pinto beans, peas, peanuts) are combinations that provide complete proteins. Eggs are not consumed by vegans. Fruits and vegetables alone will not provide the essential amino acids. Vitamin and mineral supplements do not provide amino acids.

The physician diagnoses a 3-cm cyst in the ovary of a 28-year-old patient. You expect the initial treatment to include a. initiating hormone therapy. b. scheduling a laparoscopy to remove the cyst. c. examining the patient after her next menstrual period. d. aspirating the cyst and sending the fluid to pathology.

ANS: C Most ovarian cysts regress spontaneously. Cysts in women of childbearing age may decrease within one cycle, so treatment is not necessary at this point. It is too early to anticipate removal of the cysts. Most ovarian cysts regress spontaneously within one cycle. A transvaginal ultrasound examination will help determine if the cyst is fluid-filled or solid. The cyst can then be removed if warranted.

When explaining the recommended weight gain to your patient, the nurse's teaching should include which statement? a. "All pregnant women need to gain a minimum of 25 to 35 lb." b. "The fetus, amniotic fluid, and placenta require 15 lb of weight gain." c. "Weight gain in pregnancy is based on the patient's prepregnant body mass index." d. "More weight should be gained in the first and second trimesters and less in the third."

ANS: C Recommendations for weight gain in pregnancy are based on the woman's prepregnancy weight for her height (body mass index). Depending on the prepregnant weight, recommendation for weight gain may be more or less than 25 to 35 lb. The combination of the fetus, amniotic fluid, and placenta averages about 11 lb in the patient who has a normal BMI. Less weight should be gained in the first trimester, when the fetus needs fewer nutrients for growth, and more in the third trimester, when fetal growth is accelerated.

Some of the embryo's intestines remain within the umbilical cord during the embryonic period because the a. intestines need this time to grow until week 15. b. nutrient content of the blood is higher in this location. c. abdomen is too small to contain all the organs while they are developing. d. umbilical cord is much larger at this time than it will be at the end of pregnancy.

ANS: C The abdominal contents grow more rapidly than the abdominal cavity, so part of their development takes place in the umbilical cord. By 10 weeks, the abdomen is large enough to contain them. The intestines remain within the umbilical cord only until about week 10. Blood supply is adequate in all areas; intestines stay in the umbilical cord for about 10 weeks because they are growing faster than the abdomen. Intestines begin their development within the umbilical cord, but only because the liver and kidneys occupy most of the abdominal cavity, not because of the size of the umbilical cord.

Which treatment option minimizes the development of lymphedema in the surgical management of a patient with breast cancer? a. Radical mastectomy procedure b. Radiation therapy c. Sentinel lymph node mapping d. Ultrasound

ANS: C The use of sentinel lymph node mapping identifies only those affected lymph node tissues that require surgical removal and helps minimize the development of lymphedema in the surgical management of a patient with breast cancer. Radical mastectomy is associated with lymphedema in the postsurgical breast cancer patient because of the removal of lymph node tissue. Radiation therapy is not associated with a decrease in lymphedema for the breast cancer patient. Ultrasound as an intervention does not affect the development of lymphedema.

The method of contraception that is considered the safest for women is a(n) a. IUD. b. diaphragm. c. male condom. d. oral contraceptive

ANS: C A male condom does not have any side effects or risk factors for the woman. Oral contraception, an IUD, and the diaphragm all have significant side effects or risk factors for the woman.

The drug of choice to treat a gonorrhea infection is a. penicillin G (Pfizerpen). b. tetracycline (Achromycin). c. ceftriaxone (Rocephin). d. acyclovir (Zovirax).

ANS: C Additional drugs include cefixime (Suprate) or ciprofloxacin (Cipro). Ceftriaxone is effective for treatment of all gonococcal infections. Penicillin G is used most commonly to treat syphilis. Tetracycline is prescribed to treat chlamydial infections. Acyclovir is most commonly used to treat herpes genitalis.

Which contraceptive method provides protection against sexually transmitted diseases? a. Oral contraceptives b. Tubal ligation c. Male or female condoms d. Intrauterine device (IUD)

ANS: C Because latex condoms provide the best protection available, they should be used during any potential exposure to a sexually transmitted disease (STD). Only the barrier methods provide some protection from STDs. A tubal ligation is considered a permanent contraceptive method; however, does not offer any protection against sexually transmitted diseases. IUDs are inserted into the uterus and provide no protection from STDs.

During vital sign assessment of a pregnant patient in her third trimester, the patient complains of feeling faint, dizzy, and agitated. Which nursing intervention is most appropriate? a. Have the patient stand up and retake her blood pressure. b. Have the patient sit down and hold her arm in a dependent position. c. Have the patient turn to her left side and recheck her blood pressure in 5 minutes. d. Have the patient lie supine for 5 minutes and recheck her blood pressure on both arms.

ANS: C Blood pressure is affected by positioning during pregnancy. The supine position may cause occlusion of the vena cava and descending aorta. Turning the pregnant woman to a lateral recumbent position alleviates pressure on the blood vessels and quickly corrects supine hypotension. Pressures are significantly higher when the patient is standing. This would cause an increase in systolic and diastolic pressures. The arm should be supported at the same level of the heart. The supine position may cause occlusion of the vena cava and descending aorta, creating hypotension.

A patient, gravida 2, para 1, comes for a prenatal visit at 20 weeks of gestation. Her fundus is palpated 3 cm below the umbilicus. This finding is a. appropriate for gestational age. b. a sign of impending complications. c. lower than normal for gestational age. d. higher than normal for gestational age.

ANS: C By 20 weeks, the fundus should reach the umbilicus. The fundus should be at the umbilicus at 20 weeks, so 3 cm below the umbilicus is an inappropriate height and needs further assessment. This is lower than expected at this date. It may be a complication, but it may also be because of incorrect dating of the pregnancy.

The patient who has had an intrauterine device (IUD) inserted should be instructed to a. use a vinegar douche weekly for 4 weeks. b. have the IUD replaced every 2 to 4 years. c. check the placement of the string once a week for 4 weeks. d. use another method of contraception for 2 weeks after insertion.

ANS: C Checking the placement of the string is necessary to determine whether the IUD is still correctly positioned. If the string is shorter or longer than when checked previously, the patient should contact her physician. A vinegar douche weekly for 4 weeks is not necessary. An IUD can be left in place for up to 10 years. A second method of contraception is not required after insertion of the IUD; it is effective immediately

A gravida 1 patient at 32 weeks of gestation reports that she has severe lower back pain. What should the nurse's assessment include? a. Palpation of the lumbar spine b. Exercise pattern and duration c. Observation of posture and body mechanics d. Ability to sleep for at least 6 hours uninterrupted

ANS: C Correct posture and body mechanics can reduce lower back pain caused by increasing lordosis. Pregnancy should not cause alterations in the spine. Any assessment for malformation should be done early in pregnancy. Certain exercises can help relieve back pain. Rest is important for overall well-being; however, the primary concern related to back pain is a thorough evaluation of posture and body mechanics.

The nurse is conducting a staff in-service on multifetal pregnancy. Which statement regarding dizygotic twin development should the nurse include in the teaching session? a. Dizygotic twins arise from two fertilized ova and are the same sex. b. Dizygotic twins arise from a single fertilized ovum and are always of the same sex. c. Dizygotic twins arise from two fertilized ova and may be the same sex or different sexes. d. Dizygotic twins arise from a single fertilized ovum and may be the same sex or different sexes.

ANS: C Dizygotic twins arise from two ova that are fertilized by different sperm. They may be the same or different gender, and they may not have similar physical traits. Monozygotic twins are always the same sex. A single fertilized ovum that produces twins is called monozygotic. Dizygotic twins are from two fertilized ova and may or may not be the same sex.

Which information is covered by early pregnancy classes offered in the first and second trimesters? a. Methods of pain relief b. The phases and stages of labor c. Coping with common discomforts of pregnancy d. Prebirth and postbirth care of a patient having a cesarean birth

ANS: C Early pregnancy classes focus on the first two trimesters and cover information on adapting to pregnancy, dealing with early discomforts, and understanding what to expect in the months ahead. Methods of pain relief are discussed in a childbirth preparation class. The phases and stages of labor are usually covered in a childbirth preparation class. Cesarean birth preparation classes discuss prebirth and postbirth of a patient having a cesarean birth.

A patient at 8 weeks' gestation complains to the nurse, "I feel sick almost every morning. And I throw up at least two or three times a week." What is the nurse's best guidance for this patient? a. "Do you like cheese?" b. "Try eating four meals a day instead of three meals a day." c. "Try eating peanut butter on whole wheat bread right before going to bed." d. "If you can eat enough throughout the day, you don't have to worry about being sick."

ANS: C Eating a bedtime protein snack helps maintain glucose levels throughout the night. Cheese is high in fat and can aggravate nausea. Small and frequent meals is the optimal recommendation. Four meals a day would not be ideal for a patient experiencing nausea, she needs to eat more frequently. Consumption is not the patient's stated concern—it is the nausea and vomiting

Informed consent concerning contraceptive use is important since some of the methods a. may not be reliable. b. require a surgical procedure to insert. c. have potentially dangerous side effects. d. are invasive procedures that require hospitalization.

ANS: C It is important for couples to be aware of potential side effects in order for them to make an informed decision regarding the use of contraceptives. Even if a method is less reliable, it still carries with it side effects that require informed consent. A written consent is required if the contraceptive choice involves a surgical procedure. Some contraceptive procedures are invasive; however, do not require hospitalization for insertion

A patient is 27 years old and delivered her first baby yesterday. She and her husband do not want to have another baby for at least 3 to 4 years. The most appropriate method of birth control to meet their needs is a. withdrawal. b. fertility awareness method. c. combination of condoms and foam. d. vasectomy with a reversal in 3 years

ANS: C Of the methods listed, condoms and foam would be the best for this couple. Withdrawal is the least effective form of birth control. Fertility awareness is not that effective and an unwanted pregnancy could result. They want another child, therefore a vasectomy with a reversal would not be an appropriate option

An infant is diagnosed with fetal anemia. Which information would support this clinical diagnosis? a. Presence of excess maternal hormones b. Maternal blood type O-negative, Rh-negative, and infant blood type O-negative, Rh-negative c. Passive immunity d. Rh-negative mother and Rh-positive baby

ANS: C Passive immunity provides temporary protection to the baby based on the transfer of maternal antibodies. Maternal hormones would not lead to a clinical diagnosis of fetal anemia. These blood types and Rh factors are the same; therefore, no antibodies will be created. In this situation, an Rh-negative mother and Rh-positive baby will result in stimulation of antibodies that will stimulate a reaction leading to hemolysis.

A patient with an IUD in place has a positive pregnancy test. When planning care, the nurse will base decisions on which anticipated action? a. A therapeutic abortion will need to be scheduled since fetal damage is inevitable. b. Hormonal analyses will be done to determine the underlying cause of the false-positive test result. c. The IUD will need to be removed to avoid complications such as miscarriage or infection. d. The IUD will need to remain in place to avoid injuring the fetus.

ANS: C Pregnancy with an intrauterine device (IUD) in place is unusual; however, it can occur and cause complications such as spontaneous abortion and infection. A therapeutic abortion is not indicated unless infection occur

A woman undergoing evaluation of infertility states, "At least when we're through with all of these tests, we will know what is wrong." The nurse's best response is a. "I know the test will identify what is wrong." b. "I'm sure that once you finish these tests, your problem will be resolved." c. "Even with diagnostic testing, infertility remains unexplained in about 20% of couples." d. "Once you've identified your problem, you may want to look at the option of adoption."

ANS: C Problems with infertility must be approached realistically. Nurses should not make judgments or provide false reassurance. Providing accurate information to the couple is the optimal response. The nurse should not make statements indicating that problems will be resolved, because this gives a false impression. The tests are not always definitive; therefore, the nurse should not offer her view or opinion; rather, should state the facts.

A patient relates a story of how her boyfriend is feeling her aches and pains associated with her pregnancy. She is concerned that her boyfriend is making fun of her concerns. How would you respond to this patient statement? a. Tell her not to worry because it is natural for her boyfriend to make her feel better by identifying with her pregnancy. b. Refer the patient to a psychologist for counseling to deal with this problem because it is clearly upsetting her. c. Explain that her boyfriend may be experiencing couvade syndrome and that this is a normal finding seen with male partners. d. Ask the patient specifically to define her concerns related to her relationship with her boyfriend and suggest methods to stop this type of behavior by her significant other.

ANS: C Provide factual information that will help reduce stress and modify acceptance. Telling her not to worry does not address the possibility that her boyfriend may be experiencing couvade syndrome. The patient is expressing concern but does not have all the facts related to couvade syndrome and requires education, rather than referral. Couvade syndrome is not an abnormal condition and should be treated with acceptance and understanding.

What is the best explanation that the nurse can provide to a patient who is concerned that she has "pseudoanemia" of pregnancy? a. Have her write down her concerns and tell her that you will ask the physician to respond once the lab results have been evaluated. b. Tell her that this is a benign self-limiting condition that can be easily corrected by switching to a high-iron diet. c. Inform her that because of the pregnancy, her blood volume has increased, leading to a substantial dilution effect on her serum blood levels, and that most women experience this condition. d. Contact the physician and get a prescription for iron pills to correct this condition

ANS: C Providing factual information based on physiologic mechanisms is the best option. Although having the patient write down her concerns is reasonable, the nurse should not refer this conversation to the physician but rather address the patient's specific concerns. Switching to a high-iron diet will not correct this condition. This physiologic pattern occurs during pregnancy as a result of hemodilution from excess blood volume. Iron medication is not indicated for correction of this condition. There is no need to contact the physician for a prescription

The health care provider reports that the primigravida's fundus can be palpated at the umbilicus. Which priority question will the nurse include in the patient's assessment? a. "Have you noticed that it is easier for you to breathe now?" b. "Would you like to hear the baby's heartbeat for the first time?" c. "Have you felt a fluttering sensation in your lower pelvic area yet?" d. "Have you recently developed any unusual cravings, such as for chalk or dirt?"

ANS: C Quickening is the first maternal sensation of fetal movement and is often described as a fluttering sensation. Quickening is detected at approximately 20 weeks in the primigravida and as early as 16 weeks in the multigravida. The fundus is at the umbilicus at 20 weeks' gestation. Lightening is associated with descent of the fetal head into the maternal pelvis and is associated with improved lung expansion. Lightening occurs approximately 2 weeks before birth in the primipara. Fetal heart tones can be detected by Doppler as early as 9 to 12 weeks of gestation. Pica is the craving for nonnutritive substances such as chalk, dirt, clay, or sand. It can develop at any time during pregnancy. It can be associated with malnutrition and the health care provider should monitor the patient's hematocrit/hemoglobin, zinc, and iron levels.

Chromosome analysis is a diagnostic test that should be offered to which couple? a. Never conceived b. Has long-standing infertility c. Has had repeated pregnancy losses d. Has a normal child but has not conceived again

ANS: C Repeated failures to carry a pregnancy to term may indicate genetic defects in the fetus that are incompatible with life. A couple that has never conceived would not be offered chromosome analysis. Long-standing infertility is not an indicator for chromosome analysis. Secondary infertility with an existing normal child would not be an indicator for chromosome analysis.

What is the gravida and para for a patient who delivered triplets 2 years ago and is now pregnant again? a. 2, 3 b. 1, 2 c. 2, 1 d. 1, 3

ANS: C She has had two pregnancies (gravida 2); para refers to the outcome of the pregnancy rather than the number of infants from that pregnancy. She is pregnant now, so that would make her a gravida 2. She is para 1 because she had one pregnancy that progressed to the age of viability.

When using the basal body temperature method of family planning, the woman should understand that a. she will remain fertile for 5 days after ovulation. b. she should take her temperature each night before going to bed. c. her temperature will increase about 0.2 to 0.4°C (0.4 to 0.8°F) after ovulation. d. her temperature is normally lower during the second half of her cycle.

ANS: C The basal body temperature will increase approximately 0.2 to 0.4°C (0.4 to 0.8°F) at the same time ovulation occurs. The woman is fertile for up to 18 days. She should take her temperature at the same time upon rising in the morning. A woman's temperature is usually higher in the second half of her cycle.

Which contraceptive method should be contraindicated in a patient with a history of toxic shock syndrome? a. Condom b. Spermicide c. Cervical cap d. Oral contraceptives

ANS: C The cervical cap should not be used in women with a history of toxic shock syndrome. A condom, spermicide, or oral contraceptives are not contraindicated with a history of toxic shock syndrome

Which of the following statements is correct regarding the use of contraception and the occurrence of sexually transmitted diseases (STDs)? a. As long as the oral contraception method is used correctly, there is no transmission of STDs during sexual activity. b. Oral contraceptives provide the greatest protection against getting STDs. c. Barrier methods, if used correctly, are more likely to protect individuals from STDs as compared with other contraceptive methods. d. It is less likely to see transmission of STDs if patients engage in oral sex as opposed to vaginal penetration.

ANS: C The correct use of barrier methods helps protect against the transmission of STDs compared with other methods of contraception. The use of oral contraceptives has no effect on the transmission of STDs. The effectiveness of oral contraceptives is increased related to the prevention of pregnancy compared with other methods with the exception of abstinence. The method of sexual activity does not affect the transmission of STDs.

The major difference between the diaphragm and the cervical cap is that the diaphragm a. is more effective. b. requires spermicide. c. applies pressure on the urethra. d. has no contribution to toxic shock syndrome.

ANS: C The diaphragm is made to fit snugly in the vaginal area and contains a hard rim that may exert pressure on the urethra. The cervical cap is smaller and fits around the cervix. The cervical cap is not more effective than a diaphragm. The diaphragm does not require spermicide. Both methods may contribute to the onset of toxic shock syndrome.

An expectant father asks the nurse, "Which part of the mature sperm contains the male chromosome?" What is the correct response by the nurse? a. X-bearing sperm b. The tail of the sperm c. The head of the sperm d. The middle portion of the sperm

ANS: C The head of the sperm contains the male chromosomes that will join the chromosomes of the ovum. If an X-bearing sperm fertilizes the ovum, the baby will be female. The tail of the sperm helps propel the sperm toward the ovum. The middle portion of the sperm supplies energy for the tail's whip-like action.

Which patient has correctly increased her caloric intake from her recommended pregnancy intake to the amount necessary to sustain breastfeeding in the first 6 postpartum months? a. From 1800 to 2200 calories per day b. From 2000 to 2500 calories per day c. From 2200 to 2530 calories per day d. From 2500 to 2730 calories per day

ANS: C The increased calories necessary for breastfeeding are 500, with 330 calories coming from increased caloric intake and 170 calories from maternal stores. An increase of 230 calories is insufficient for breastfeeding. An increase of 400 and 500 calories is above the recommended amount.

The nurse is conducting a prenatal nutrition education class for a group of nursing students. Which statement best describes the condition known as pica? a. Iron-deficiency anemia b. Intolerance to milk products c. Ingestion of nonfood substances d. Episodes of anorexia and vomiting

ANS: C The practice of eating substances not normally thought of as food is called pica. Clay, dirt, and solid laundry starch are the substances most commonly ingested. Pica may produce iron-deficiency anemia if proper nutrition is decreased. Intolerance to milk products is termed lactose intolerance. Pica is not related to anorexia and vomiting.

In reviewing genetic testing for a female patient, you note the presence of BRCA1, BRCA2, and CHEK2. How should these findings be interpreted? a. There is no increased likelihood that the patient will develop breast or ovarian cancer. b. There is an increased likelihood only for the development of breast cancer in a woman. c. More information is needed to interpret these findings based on the patient's family history and the patient's current and past medical history. d. A radical bilateral mastectomy is required as soon as possible, since the cancer may have already undergone sub-metastasis.

ANS: C The presence of genetic markers (BRCA1, BRCA2, and CHEK2) provides strong indicators of the increased risk for the development of breast cancer in males and females as well as ovarian cancer. It is important to obtain additional information in order that a treatment plan can be developed and implemented to improve patient outcomes. There is an increased likelihood that the patient will develop breast or ovarian cancer. Stating that there is an increased likelihood only for the development of breast cancer in a woman fails to include that men are also at risk of developing breast cancer. At this point, surgical intervention is speculative because the presence of biomarkers does not indicate that sub-metastasis has occurred or that the cancer has even developed

Large amounts of leukocytes in the seminal fluid suggest a clinical finding of a. inadequate fructose. b. inflammation of the testes. c. an infection of the genital tract. d. an obstruction in the vas deferens.

ANS: C The presence of large amounts of leukocytes suggests an infection. Adequate fructose must be present to supply energy for the sperm. An inflammatory process would be diagnosed by abnormal consistency or chemical composition. If an obstruction is present, the total amount of seminal fluid would be decreased

A pregnant woman complains of frequent heartburn. The patient states that she has never had these symptoms before and wonders why this is happening now. The most appropriate response by the nurse is to a. examine her dietary intake pattern and tell her to avoid certain foods. b. tell her that this is a normal finding during early pregnancy and will resolve as she gets closer to term. c. explain to the patient that physiologic changes caused by the pregnancy make her more likely to experience these types of symptoms. d. refer her to her health care provider for additional testing because this is an abnormal finding.

ANS: C The presentation of heartburn is a normal abnormal finding that can occur in pregnant woman because of relaxation of the lower esophageal sphincter as a result of the physiologic effects of pregnancy. Although foods may contribute to the heartburn, the patient is asking why this presentation is occurring, so the nurse should address the cause first. It is independent of gestation. There is no need to refer to the physician at this time because this is a normal abnormal finding. There is no evidence of complications ensuing from this presentation.

What is the physiologic reason for vascular volume increasing by 40% to 60% during pregnancy? a. Prevents maternal and fetal dehydration b. Eliminates metabolic wastes of the mother c. Provides adequate perfusion of the placenta d. Compensates for decreased renal plasma flow

ANS: C The primary function of increased vascular volume is to transport oxygen and nutrients to the fetus via the placenta. Preventing maternal and fetal dehydration is not the primary reason for the increase in volume. Assisting with pulling metabolic wastes from the fetus for maternal excretion is one purp

One of the assessments performed in the birth room is checking the umbilical cord for blood vessels. Which finding is considered to be within normal limits? a. One artery and one vein b. Two veins and one artery c. Two arteries and one vein d. Two arteries and two veins

ANS: C The umbilical cord contains two arteries and one vein to transport blood between the fetus and the placenta. Any option other than two arteries and one vein is considered abnormal and requires further assessment. Two veins and one artery is abnormal and may indicate an anomaly. Two arteries instead is a normal finding; this infant would require further assessment for anomalies due to the finding of two veins.

Which sexually transmitted disease can be treated and eradicated? a. Herpes b. AIDS c. Chlamydia d. Venereal warts

ANS: C Treatment options chlamydial bacterial infection include: azithromycin, doxycycline, ofloxacin, levofloxacin, or erythromycin. Concurrent treatment of all sexual partners is necessary to prevent recurrence. Because no cure is known for herpes, treatment focuses on pain relief and preventing secondary infections. Because no cure is known for AIDS, prevention and early detection are the main focus. Condylomata acuminata is caused by the human papillomavirus (HPV). No treatment eradicates the virus; however, there is a vaccine available.

A patient with a history of a cystocele should contact the physician if she experiences a. backache. b. constipation. c. urinary frequency and burning. d. involuntary loss of urine when she coughs

ANS: C Urinary frequency and burning are symptoms of cystitis, a common problem associated with cystocele. Back pain is a symptom of uterine prolapse. Constipation may be a problem for the patient with a rectocele. Involuntary loss of urine during coughing is referred to as stress incontinence and is not an emergency.

Physiologic anemia often occurs during pregnancy due to a. inadequate intake of iron. b. the fetus establishing iron stores. c. dilution of hemoglobin concentration. d. decreased production of erythrocytes.

ANS: C When blood volume expansion is more pronounced and occurs earlier than the increase in red blood cells, the woman will have physiologic anemia, which is the result of dilution of hemoglobin concentration rather than inadequate hemoglobin. Inadequate intake of iron may lead to true anemia. If the woman does not take an adequate amount of iron, true anemia may occur when the fetus pulls stored iron from the maternal system. There is increased production of erythrocytes during pregnancy.

In reviewing information related to the occurrence of pregnancies using a focus group discussion with women, concern was expressed that many of them had problems using their respective type of contraception. As a result of noncompliance issues several women became pregnant. Based on this information, the nurse would incorporate which of the following in a teaching plan for group members? a. Provide information relative to product recalls of contraceptive devices. b. Have the patients keep a contraceptive diary related to the consistency of using methods because it is apparent that they forgot to use their preferred method as directed. c. Have the patients consider switching to a different form of contraception because the contraception did not prevent pregnancy for them. d. Plan for assessing the patients' knowledge related to the contraception methods and provide information to increase the knowledge base so that the effectiveness rate would improve

ANS: D A typical effectiveness rate refers to the occurrence of pregnancy while using a specific contraception method. If contraception is used correctly and consistently, pregnancy should not occur. A decreased effectiveness rate is associated with incorrect usage in terms of application or inconsistent use. Providing information relative to product recalls of contraceptive devices refers to an ideal effectiveness rate; the implication is that the contraception method, although used correctly, is at fault. There is no evidence to support this finding. Having the patient keep a contraceptive diary does not address the primary concerns related to the typical effectiveness rate. Having the patient switch methods may not be necessary because the primary focus is to determine the knowledge base and identify learning needs.

In order to increase the absorption of iron by a pregnant patient, which beverage should an iron preparation be given with? a. Tea b. Milk c. Coffee d. Orange juice

ANS: D Vitamin C source may increase the absorption of iron and would be the optimal choice. Tannin in the tea reduces the absorption of iron. The calcium and phosphorus in milk decrease iron absorption. Decreased intake of caffeine is recommended during pregnancy.

Which patient would require additional calories and nutrients? a. A 36-year-old female gravida 2, para 1, in her first trimester of pregnancy b. An 18-year-old female who delivered a 7-lb baby and is bottle feeding c. A 23-year-old female who had a cesarean birth and is bottle feeding d. A 20-year-old female who had a vaginal birth 5 months ago and is breastfeeding

ANS: D A patient who is breastfeeding will require more calories and nutrients than women who are pregnant. The type of birth has no impact on nutrient intake. A patient who is bottle-feeding does not require additional calories.

When documenting a patient encounter, which term will the nurse use to describe the woman who is in the 28th week of her first pregnancy? a. Multigravida b. Multipara c. Nullipara d. Primigravida

ANS: D A primigravida is a woman pregnant for the first time. A multigravida has been pregnant more than once. A nullipara is a woman who has never been pregnant or has not completed a pregnancy of 20 weeks or more. A primipara has delivered one pregnancy of at least 20 weeks. A multipara has delivered two or more pregnancies of at least 20 weeks.

Which comment made by a patient in her first trimester indicates ambivalent feelings? a. "My body is changing so quickly." b. "I haven't felt well since this pregnancy began." c. "I'm concerned about the amount of weight I've gained." d. "I wanted to become pregnant, but I'm scared about being a mother."

ANS: D Ambivalence refers to conflicting feelings. Expressing a concern about being a mother indicates possible ambivalent feelings. Not feeling well since the pregnancy began does not reflect conflicting feelings. The woman is trying to confirm the pregnancy when she is stating the rapid changes to her body. She is not expressing conflicting feelings. By expressing concerns over gaining weight, which is normal, the woman is trying to confirm the pregnancy.

The nurse is explaining the function of the placenta to a pregnant patient. Which statement indicates to the nurse that further clarification is necessary? a. "My baby gets oxygen from the placenta." b. "The placenta functions to help excrete waste products." c. "The nourishment that I take in passes through the placenta." d. "The placenta helps maintain a stable temperature for my baby."

ANS: D Amniotic fluid and not the placenta helps with thermoregulation. The remaining statements are correct regarding placental function.

A patient has been diagnosed with an incompetent cervix (the cervix will not remain closed). What treatment option will be incorporated into the plan of care for this patient? a. Bed rest throughout the pregnancy b. Wait and see approach to determine if the patient goes into preterm labor c. Preparation for cerclage procedure at 32 weeks' gestation d. More frequent ultrasounds to assess progression of pregnancy

ANS: D An incompetent cervix would place the patient in a high-risk category, and more frequent ultrasound monitoring would be included in her care plan. Although bed rest may be ordered, there is conflicting evidence regarding the merits of this intervention. An incompetent cervix is considered a clinical abnormality, therefore the standard of care requires appropriate surgical intervention. A cerclage procedure is typically done much earlier in the pregnancy period.

Which suggestion is most helpful for the pregnant patient who is experiencing heartburn? a. Drink plenty of fluids at bedtime. b. Eat only three meals a day so the stomach is empty between meals. c. Drink coffee or orange juice immediately on arising in the morning. d. Use Tums or Rolaids to obtain relief, as directed by the health care provider

ANS: D Antacids high in calcium (e.g., Tums, Rolaids) can provide temporary relief. Fluids overstretch the stomach and may precipitate reflux when lying down. Instruct the patient to eat five or six small meals per day rather than three full meals. Coffee and orange juice stimulate acid formation in the stomach and may need to be eliminated from the diet.

A pregnant woman has come to the emergency department with complaints of nasal congestion and epistaxis. Which is the correct interpretation of these symptoms by the health care provider? a. Nasal stuffiness and nosebleeds are caused by a decrease in progesterone. b. These conditions are abnormal. Refer the patient to an ear, nose, and throat specialist. c. Estrogen relaxes the smooth muscles in the respiratory tract, so congestion and epistaxis are within normal limits. d. Estrogen causes increased blood supply to the mucous membranes and can result in congestion and nosebleeds.

ANS: D As capillaries become engorged, the upper respiratory tract is affected by the subsequent edema and hyperemia, which causes these conditions, seen commonly during pregnancy. Progesterone is responsible for the heightened awareness of the need to breathe in pregnancy. Progesterone levels increase during pregnancy. The patient should be reassured that these symptoms are within normal limits. No referral is needed at this time. Relaxation of the smooth muscles in the respiratory tract is affected by progesterone.

Findings of a Pap smear exam denote atypical cells of undetermined significance (ASCUS). The Pap test is repeated at 6 months and the same finding of ASCUS is reported. Which therapeutic treatment option would the nurse expect the practitioner to order? a. Mammography b. Bone scan c. Transvaginal ultrasound d. Biopsy

ANS: D Based on the standard of care, a colposcopy or biopsy of the cervix is indicated. A Pap smear is performed to evaluate the cervix. There is no indication that mammography, which is used to assess and evaluate breast tissue, is required. There is no evidence to warrant a bone scan. Although a transvaginal ultrasound might be included in the treatment plan, the Pap smear indicates cervical pathology so a colposcopy or biopsy is indicated.

Which option could be used for the treatment and management of a patient who reports mild pain associated with a clinical diagnosis of fibrocystic breast disease? a. Chamomile tea as a relaxant therapy b. Danazol (Danocrine) c. Tamoxifen (Nolvadex) d. Over-the-counter nonsteroidal antiinflammatory drug (NSAID) therapy

ANS: D Because the patient is reporting mild pain, NSAIDs may provide adequate pain relief and comfort. It is recommended that tea, coffee, and/or other stimulants be limited or restricted for patients with fibrocystic breast disease. Danazol is typically used for moderate to severe pain for patients with fibrocystic breast disease because its use is associated with more serious side effects. Danazol should not be used longer than 4 to 6 months. The patient reports mild pain so this would not be warranted. Tamoxifen is a selective estrogen receptor modulator (SERM) used for the treatment of breast cancers and also for moderate to severe pain in fibrocystic breast disease. The patient reports mild pain, so this would not be warranted

A patient in her third trimester of pregnancy is asking about safe travel. Which statement should the nurse provide regarding safe travel during pregnancy? a. "Only travel by car during pregnancy." b. "Avoid use of the seat belt during the third trimester." c. "You can travel by plane until your 38th week of gestation." d. "If you are traveling by car stop to walk every 1 to 2 hours."

ANS: D Car travel is safe during normal pregnancies. Suggest that the woman stop to walk every 1 to 2 hours so she can empty her bladder. Walking also helps decrease the risk of thrombosis that is elevated during pregnancy. Seat belts should be worn throughout the pregnancy. Instruct the woman to fasten the seat belt snugly, with the lap belt under her abdomen and across her thighs and the shoulder belt in a diagonal position across her chest and above the bulge of her uterus. Travel by plane is generally safe up to 36 weeks if there are no complications of the pregnancy, so only traveling by car is an inaccurate statement.

The nurse is meeting with a patient with an elevated BMI regarding an optimal diet for pregnancy. Which major source of nutrients should be a significant component of this patient's diet? a. Fats b. Fiber c. Simple sugars d. Complex carbohydrates

ANS: D Complex carbohydrates supply the pregnant woman with vitamins, minerals, and fiber. Fats provide 9 calories in each gram, in contrast to carbohydrates and proteins, which provide only 4 calories in each gram. Fiber is supplied primarily by complex carbohydrates. The most common simple carbohydrate is table sugar, which is a source of energy but does not provide any nutrients.

Which physiologic adaptation of pregnancy may lead to increased constipation during the pregnancy? a. Increased emptying time in the intestines b. Abdominal distention and bloating c. Decreased absorption of water d. Decreased motility in the intestines

ANS: D Decreased motility in the intestines leading to increased water absorption would cause constipation. Increased emptying time in the intestines leads to increased nutrient absorption. Abdominal distention and bloating are a result of increased emptying time in the intestines. Decreased absorption of water would not cause constipation.

Which is the most important reason for evaluating the pattern of weight gain in pregnancy? a. Prevents excessive adipose tissue deposits b. Determines cultural influences on the woman's diet c. Assesses the need to limit caloric intake in obese women d. Identifies potential nutritional problems or complications of pregnancy

ANS: D Deviations from the recommended pattern of weight gain may indicate nutritional problems or developing complications. Excessive adipose tissue may occur with excess weight gain but is not the reason for monitoring the weight gain pattern. The pattern of weight gain is not affected by cultural influences. It is important to monitor the pattern of weight gain for the developing complications.

Which patient is a safe candidate for the use of oral contraceptives? a. 39-year-old with a history of thrombophlebitis. b. 16-year-old with a benign liver tumor. c. 20-year-old who suspects she may be pregnant. d. 43-year-old who does not smoke cigarettes.

ANS: D Heavy cigarette smoking is a contraindication to oral contraceptive use due to risk of stroke. Oral contraceptives are contraindicated with a history of thrombophlebitis. Liver tumors, benign or malignant, preclude the use of oral contraceptives. Pregnancy is also a contraindication

A nurse is leading a discussion regarding options for birth control. Which of the following methods is considered the most reliable? a. Coitus interruptus b. Breastfeeding c. Natural family planning d. Intrauterine device

ANS: D IUDs are 99% effective. Although coitus interruptus is considered 78% effective, it requires great control by the man and may be unsatisfying for both partners. Women who exclusively breast feed (without formula or solid food supplementation) may avoid ovulation. Another method of birth control should be used when the frequency of breastfeeding decreases. At 6 months postpartum ovulation commences in most women, even if exclusively breastfeeding. Couples must be highly motivated to use natural family planning because they must abstain from sex for as much as half of the menstrual cycle.

Which clinical finding is associated with inadequate maternal weight gain during pregnancy? a. Prolonged labor b. Preeclampsia c. Gestational diabetes d. Low-birth-weight infant

ANS: D Inadequate maternal weight gain during pregnancy can manifest in the birth of a low-birth-weight infant. Prolonged labor and gestational diabetes are associated with excess weight gain during pregnancy. Preeclampsia is based on maternal hypertension, proteinuria, and edema states.

The patient has just learned that she is pregnant and overhears the gynecologist saying that she has a positive Chadwick's sign. When the patient asks the nurse what this means, how would the nurse respond? a. "Chadwick's sign signifies an increased risk of blood clots in pregnant women because of a congestion of blood." b. "That sign means the cervix has softened as the result of tissue changes that naturally occur with pregnancy." c. "This means that a mucus plug has formed in the cervical canal to help protect you from uterine infection." d. "This sign occurs normally in pregnancy, when estrogen causes increased blood flow in the area of the cervix."

ANS: D Increasing levels of estrogen cause hyperemia (congestion with blood) of the cervix, resulting in the characteristic bluish purple color that extends to include the vagina and labia. This discoloration, referred to as Chadwick's sign, is one of the earliest signs of pregnancy. Although Chadwick's sign occurs with hyperemia (congestion with blood), the sign does not signify an increased risk of blood clots. The softening of the cervix is called Goodell's sign, not Chadwick's sign. Although the formation of a mucus plug protects from infection, it is not called Chadwick's sign.

A pregnant patient with significant iron-deficiency anemia is prescribed iron supplements. The patient explains to the nurse that she cannot take iron because it makes her nauseous. What is the best response by the nurse? a. "Iron will be absorbed more readily if taken with orange juice." b. "It is important to take this drug regardless of this side effect." c. "Taking the drug with milk may decrease your symptoms." d. "Try taking the iron at bedtime on an empty stomach."

ANS: D Iron taken at bedtime may be easier to tolerate. All the answers are true statements; however, only the option that states that iron taken at bedtime may be easier to tolerate addresses both optimal absorption of iron and alleviation of nausea, which will not be noticeable during sleep. It is true that taking iron with milk will decrease the symptoms; however, it will also decrease absorption.

. Which comment made by a new mother exhibits understanding of her toddler's response to a new sibling? a. "I can't believe he is sucking his thumb again." b. "He is being difficult and I don't have time to deal with him." c. "When we brought the baby home, we made Michael stop sleeping in the crib." d. "My husband is going to stay with the baby so I can take Michael to the park tomorrow."

ANS: D It is important for a mother to seek time alone with her toddler to reassure him that he is loved. It is normal for a child to regress when a new sibling is introduced into the home. The toddler may have feelings of jealousy and resentment toward the new baby taking attention away from him. Frequent reassurance of parental love and affection is important. Changes in sleeping arrangements should be made several weeks before the birth so the child does not feel displaced by the new baby.

. A male patient asks, "Why do I have to use another contraceptive? I had a vasectomy last week." The best response is a. "A vasectomy is only 80% effective." b. "A vasectomy is not effective in all men." c. "Semen may contain sperm for 6 months following a vasectomy." d. "Complete sterilization doesn't occur until all sperm have left the system."

ANS: D It may take a month or longer for all sperm to be removed from the system. During that time, an additional method of contraception must be used. A vasectomy is 99% effective. Vasectomies have a high success rate; however, it may take 1 month for all the sperm to be removed from the system.

A patient who is 16 weeks pregnant with her first baby asks how long it will be before she feels the baby move. Which is the nurse's best answer? a. "You should have felt the baby move by now." b. "The baby is moving, but you can't feel it yet." c. "Some babies are quiet and you don't feel them move." d. "Within the next month you should start to feel fluttering sensations."

ANS: D Maternal perception of fetal movement (quickening) usually begins between 17 and 20 weeks after conception. Because this is her first pregnancy, movement is felt toward the later part of the 17 to 20 weeks. "The baby is moving, but you can't feel it yet" may be alarming to the woman. "Some babies are quiet and you don't feel them move" is a true statement; the fetus' movements are not strong enough to be felt until 17 to 20 weeks; however, this statement does not answer the woman's concern. Fetal movement should be felt between 17 and 20 weeks; if movement is not perceptible by the end of that time, further assessment will be necessary.

A woman who is undergoing infertility testing states, "My husband won't discuss this with me. I don't think he cares about or wants a baby." The nurse's ideal response is a. "You should confront him about this." b. "He probably doesn't understand your concern." c. "Men are sometimes less eager to have children." d. "It may be harder for him to express his feelings."

ANS: D Men often internalize their feelings, which may appear to women as lack of concern or interest. Suggesting that the woman confront her husband suggests that the woman is at fault and not communicating with her husband. "He probably doesn't understand your concern" does not explain to the woman why her husband won't discuss the problem; it passes judgment on the husband. "Men are sometimes less eager to have children" does not allow the woman to express her feelings; it offers the nurse's opinion, which is not appropriate.

Which of the patient health behaviors in the first trimester would the nurse identify as a risk factor in pregnancy? a. Sexual intercourse two or three times weekly b. Moderate exercise for 30 minutes daily c. Working 40 hours a week as a secretary in a travel agency d. Relaxing in a hot tub for 30 minutes a day, several days a week

ANS: D Pregnant women should avoid activities that might cause hyperthermia. Maternal hyperthermia, particularly during the first trimester, may be associated with fetal anomalies. She should not be in a hot tub for more than 10 minutes at less than 100F. Sexual intercourse is generally safe for the healthy pregnant woman; moderate exercise during pregnancy can strengthen muscles, reduce backache and stress, and provide a feeling of well-being; working during pregnancy is acceptable as long as the woman is not continually on her feet or exposed to environmental toxins and industrial hazards.

The nurse is reviewing a list of foods high in folic acid with a patient who is considering becoming pregnant. The nurse determines that the patient understands the teaching when the patient states she will include which list of foods in her diet? a. Peaches, yogurt, and tofu b. Strawberries, milk, and tuna c. Asparagus, lemonade, and chicken breast d. Spinach, orange juice, and fortified bran flakes

ANS: D Prepregnant, the recommendation for folic acid is 800 mcg. Foods high in folic acid are dark green leafy vegetables, legumes (beans, peanuts), orange juice, asparagus, spinach, and fortified cereal and pasta. In the United States, folic acid is added to orange juice and wheat-based products.

Which statement regarding primary dysmenorrhea is most accurate? a. Primary dysmenorrhea is experienced by all women. b. It is unaffected by oral contraceptives. c. It occurs in young multiparous women. d. It may be caused by excessive endometrial prostaglandin.

ANS: D Primary dysmenorrhea is menstrual pain without identified pathology. Some women produce excessive endometrial prostaglandin during the luteal phase of the menstrual cycle. Prostaglandin diffuses into endometrial tissue and causes uterine cramping. Contrary to popular belief, primary dysmenorrhea is not experienced by all women. Oral contraceptives can be a treatment choice if cramps associated with primary dysmenorrhea are debilitating. It most often occurs in young nulliparous women

Which physiologic findings related to gallbladder function may lead to the development of gallstones during pregnancy? a. Decrease in alkaline phosphatase levels compared with nonpregnant women b. Increase in albumin and total protein as a result of hemodilution c. Hypertonicity of gallbladder tissue d. Prolonged emptying time

ANS: D Prolonged emptying time is seen during pregnancy and may lead to the development of gallstones. In pregnancy, there is a twofold to fourfold time increase in alkaline phosphatase levels as compared with those in nonpregnant woman. During pregnancy, a decrease in albumin level and total protein is seen as a result of hemodilution. Gallbladder tissue becomes hypotonic during pregnancy.

A newly married woman states, "My friend told me I would never have a baby because I had pelvic inflammatory disease when I was younger. I don't understand how that can affect whether or not I get pregnant." The nurse's most appropriate response is a. "Your friend may be right. The disease may affect your ability to conceive." b. "Pelvic inflammatory disease may damage the ovaries and prevent ovulation." c. "Your friend has been misinformed. Fallopian tube damage occurs only following gonorrhea." d. "Infection may cause scarring and obstruction of the fallopian tubes, which can prevent the fertilized egg from reaching the uterus."

ANS: D Providing the patient with accurate complete information is the best response. If untreated, pelvic inflammatory disease produces scarring and obstruction of the fallopian tube. Tubal scarring and obstruction do not occur following gonorrhea.

A relaxation technique that can be used during the childbirth experience to decrease maternal pain perception is a. using increased environmental stimulation as a method of distraction. b. restricting family and friends from visiting during the labor period to keep the patient focused on breathing techniques. c. medicating the patient frequently to reduce pain perception. d. assisting the patient in breathing methods aimed at taking control of pain perception based on the contraction pattern.

ANS: D Relaxation techniques are aimed at incorporating mind and body activities to maintain control over pain. Additional environmental stimuli may have the opposite effect and increase patient anxiety, which will affect pain perception. Restricting visitors may have the opposite effect, leading to increased anxiety because of isolation. Medicating a patient may not decrease pain perception but may place the patient at risk for adverse reactions and/or complications of pregnancy related to medications

Which situation best describes secondary infertility in a couple? a. Never conceived. b. Had repeated spontaneous abortions. c. Not conceived after 1 year of unprotected intercourse. d. Has one child but cannot conceive a second time.

ANS: D Secondary infertility occurs in couples that have conceived before; but, are unable to conceive again. Primary infertility occurs when a couple has never conceived or who has not conceived after 1 year of unprotected intercourse (6 months if the women is over age 35). Repeated spontaneous abortions are considered primary infertility (pregnancy wastage).

A pregnant patient's diet may not meet her need for folate. Which food choice is an excellent source of this nutrient? a. Chicken b. Cheese c. Potatoes d. Green leafy vegetables

ANS: D Sources of folate include green leafy vegetables, whole grains, fruits, liver, dried peas, and beans. Chicken is a good source of protein, but poor in folate. Cheese is an excellent source of calcium, but poor in folate. Potatoes contain carbohydrates and vitamins but are poor in folate.

Which is the method of childbirth that helps prevent the fear-tension-pain cycle by using slow abdominal breathing in early labor and rapid chest breathing in advanced labor? a. Bradley b. Lamaze c. Leboyer d. Dick-Read

ANS: D The Dick-Read method helps prevent the fear-tension-pain cycle by using slow abdominal breathing in early labor and rapid chest breathing in advanced labor. The Lamaze method involves concentration and conditioning to help the woman respond to contractions with relaxation to decrease pain. Viewing childbirth as a traumatic experience, the Leboyer method uses decreased light and noise to help the newborn adapt to extrauterine life more easily. The Bradley method teaches women to use abdominal muscles to increase relaxation and breath control; it emphasizes avoidance of all medications and interventions.

What does a birth plan help the parents accomplish? a. Avoidance of an episiotomy b. Determining the outcome of the birth c. Assuming complete control of the situation d. Taking an active part in planning the birth experience

ANS: D The birth plan helps the woman and her partner look at the available options and plan the birth experience to meet their personal needs. A birth plan cannot dictate the need for or avoidance of an episiotomy. The outcome of the birth is not an absolute determinant. A birth plan does not assume complete control of the situation; it allows for expanding communication. Parents who prepare a birth plan should be educated that flexibility is essential as each labor and delivery is unique and may present unexpected complications.

The nurse is explaining the process of cell division during the preembryonic period to a group of nursing students. Which statement best describes the characteristics of the morula? a. Fertilized ovum before mitosis begins b. Double layer of cells that becomes the placenta c. Flattened, disk-shaped layer of cells within a fluid-filled sphere d. Solid ball composed of the first cells formed after fertilization

ANS: D The morula is so named because it resembles a mulberry. It is a solid ball of 12 to 16 cells that develops after fertilization. The fertilized ovum is called the zygote. The placenta is formed from two layers of cells—the trophoblast, which is the other portion of the fertilized ovum, and the decidua, which is the portion of the uterus where implantation occurs. The flattened, disk-shaped layer of cells is the embryonic disk; it will develop into the body.

The role of the nurse in family planning is to a. refer the couple to a reliable physician. b. decide on the best method for the couple. c. advise couples on which contraceptive to use. d. educate couples on the various methods of contraception.

ANS: D The nurse's role is to provide information to the couple so that they can make an informed decision about family planning. The nurse can assist the couple; they do not need to be sent to a physician for contraceptive education. The nurse's role is to evaluate the couple's knowledge base and educate regarding birth control options, not to decide which is the best contraceptive method for them to utilize

Which finding is a positive sign of pregnancy? a. Amenorrhea b. Breast changes c. Fetal movement felt by the woman d. Visualization of fetus by ultrasound

ANS: D The only positive signs of pregnancy are auscultation of fetal heart tones, visualization of the fetus by ultrasound, and fetal movement felt by the examiner. Amenorrhea is a presumptive sign of pregnancy. Breast changes are a presumptive sign of pregnancy. Fetal movement is a presumptive sign of pregnancy.

A patient reports to the clinic nurse that she has not had a period in over 12 weeks, she is tired, and her breasts are sore all of the time. The patient's urine test is positive for hCG. What is the correct nursing action related to this information? a. Ask the patient if she has had any nausea or vomiting in the morning. b. Schedule the patient to be seen by a health care provider within the next 4 weeks c. Send the patient to the maternity screening area of the clinic for a routine ultrasound. d. Determine if there are any factors that might prohibit her from seeking medical care

ANS: D The patient has presumptive and probable indications of pregnancy. However, she has not sought out health care until late in the first or early in the second trimester. The nurse must assess for barriers to seeking health care, physical or emotional, because regular prenatal care is key to a positive pregnancy outcome. Asking if the patient has nausea or vomiting will only add to the list of presumptive signs of pregnancy, and this information will not add to the assessment data to determine whether the patient is pregnant. The patient needs to see a health care provider before the next 4 weeks because she is late in seeking early prenatal care. Ultrasound testing must be prescribed by a health care provider

A pregnant patient comes into the medical clinic stating that her family and friends are telling her that she is always talking about the pregnancy and nothing else. She is concerned that something is wrong with her. What psychological behavior is she exhibiting? a. Antepartum obsession b. Ambivalence c. Uncertainty d. Introversion

ANS: D The patient is exhibiting behaviors associated with introversion and/or narcissism. These are normal findings during pregnancy as long as they do not become obsessive to the exclusion of everything else. The patient is talking about the pregnancy but there is no evidence that it is affecting her perception of reality and/or ability to perform ADLs. It is normal for pregnant women to focus on the self as being of prime importance in their life initially during the pregnancy. Some women may feel ambivalent about their pregnancy, which is a normal reaction. However, this patient's behavior does not support this finding. Some women react with uncertainty at the news of being pregnant, which is a normal reaction. However, this patient's behavior does not support this finding.

Which technique would provide the best pain relief for a pregnant woman with an occiput posterior position? a. Neuromuscular disassociation b. Effleurage c. Psychoprophylaxis d. Sacral pressure

ANS: D The use of sacral pressure may provide relief for patients who are experiencing back labor. The presentation of the fetus in a posterior position indicates this. Neuromuscular dissociation is used as a conditioned response to affect pain relief based on the mother tensing one group of muscles and focusing on releasing tension in the rest of her body. Effleurage is the process of using circular massage to effect pain relief. Psychoprophylaxis is another name for the Lamaze method of prepared childbirth.

The upper uterus is the best place for the fertilized ovum to implant due to which anatomical adaptation? a. Maternal blood flow is lower. b. Placenta attaches most firmly. c. Uterine endometrium is softer. d. Developing baby is best nourished

ANS: D The uterine fundus is richly supplied with blood and has the thickest endometrium, both of which promote optimum nourishment of the fetus. The blood supply is rich in the fundus, which allows for optimal nourishment of the fetus. If the placenta attaches too deeply, it does not easily detach. Softness is not a concern with implantation; attachment and nourishment are the major concerns.

Which concern is included in the plan of care for the patient who receives HPV (human papillomavirus) vaccine? a. It is available in oral form. b. It involves a series of two injections. c. Injections should be given over a 3-month period. d. The vaccine (Gardasil) should not be given to any patient with a sensitivity to yeast

ANS: D The vaccine should not be administered to any woman who has a sensitivity to any component of the yeast family. It is available only in injection form. It is given as a series of three injections. The series of three injections should be given over a 6-month period according to Centers for Disease Control and Prevention (CDC) recommendations. Side effects of the vaccine include: headache, fever, nausea, and dizziness.

A pregnant woman notices that she is beginning to develop dark skin patches on her face. She denies using any different type of facial products as a cleansing solution or makeup. What would the priority nursing intervention be in response to this situation? a. Refer the patient to a dermatologist for further examination. b. Ask the patient if she has been eating different types of foods. c. Take a culture swab and send to the lab for culture and sensitivity (C&S). d. Let the patient know that this is a common finding that occurs during pregnancy.

ANS: D This condition is known as chloasma or melasma (mask of pregnancy) and is a result of pigmentation changes relative to hormones. It can be exacerbated by exposure to the sun. There is no need to refer to a dermatologist. Intake of foods is not associated with exacerbation of this process. There is no need for a C&S to be taken. The patient should be assured that this is a normal finding of pregnancy.

Determine the obstetric history of a patient in her fifth pregnancy who has had two spontaneous abortions in the first trimester, one infant at 32 weeks' gestation, and one infant at 38 weeks' gestation. a. G5 T1 P2 A2 L 2 b. G5 T1 P1 A1 L2 c. G5 T0 P2 A2 L2 d. G5 T1 P1 A2 L2

ANS: D This patient is in her fifth pregnancy, which is G5, she had one viable term infant (between 38 and 42 weeks' gestation), which is T1, she had one viable preterm infant (between 20 and 37 weeks' gestation), which is P1, two spontaneous abortions (before 20 weeks' gestation), which is A2, and she has two living children, which is L2.

What is the rationale for a woman in her first trimester of pregnancy to expect to visit her health care provider every 4 weeks? a. Problems can be eliminated. b. She develops trust in the health care team. c. Her questions about labor can be answered. d. The conditions of the expectant mother and fetus can be monitored.

ANS: D This routine allows for monitoring maternal health and fetal growth and ensures that problems will be identified early. All problems cannot be eliminated because of prenatal visits; however, they can be identified early. Developing a trusting relationship should be established during these visits, but that is not the primary reason. Most women do not have questions concerning labor until the last trimester of the pregnancy.

While providing education to a primiparous patient regarding the normal changes of pregnancy, what is an important information for the nurse to share regarding Braxton Hicks contractions? a. These contractions may indicate preterm labor. b. These are contractions that never cause any discomfort. c. Braxton Hicks contractions only start during the third trimester. d. These occur throughout pregnancy, but you may not feel them until the third trimester.

ANS: D Throughout pregnancy, the uterus undergoes irregular contractions called Braxton Hicks contractions. During the first two trimesters, the contractions are infrequent and usually not felt by the woman until the third trimester. Braxton Hicks contractions do not indicate preterm labor. Braxton Hicks contractions can cause some discomfort, especially in the third trimester. Braxton Hicks contractions occur throughout the whole pregnancy.

To determine cultural influences on a patient's diet, what is the nurse's primary action? a. Evaluate the patient's weight gain during pregnancy. b. Assess the socioeconomic status of the patient. c. Discuss the four food groups with the patient. d. Identify the food preferences and methods of food preparation common to the patient's culture.

ANS: D Understanding the patient's food preferences and how she prepares food will assist the nurse in determining whether the patient's culture is adversely affecting her nutritional intake. Evaluating a patient's weight gain during pregnancy should be included for all patients, not just for those who are culturally different. The socioeconomic status of the patients may alter the nutritional intake, but not the cultural influence. Teaching the food groups to the patient should come after assessing food preferences.

An expectant couple asks the nurse about intercourse during pregnancy and whether it is safe for the baby. What information should the nurse provide? a. Intercourse is safe until the third trimester. b. Safer sex practices should be used once the membranes rupture. c. Intercourse should be avoided if any spotting from the vagina occurs afterward. d. Intercourse and orgasm are often contraindicated if a history of or signs of preterm labor are present.

ANS: D Uterine contractions that accompany orgasm can stimulate labor and would be problematic if the woman is at risk for or has a history of preterm labor. Intercourse can continue as long as the pregnancy is progressing normally. Rupture of the membranes may require abstaining from intercourse. Safer sex practices are always recommended. Some spotting can normally occur as a result of the increased fragility and vascularity of the cervix and vagina during pregnancy.

When should iron supplementation during a normal pregnancy begin? a. Before pregnancy b. In the first trimester c. In the third trimester d. In the second trimester

ANS: D Vitamin supplements should be prescribed in the second trimester, when the need for iron is increased. Healthy young women do not usually need iron supplementation for their diets. Morning sickness in the first trimester increases the routine side effects of iron supplements. The iron supplements may continue to be prescribed in the third trimester and during the postpartum period.


Set pelajaran terkait

5 - Benchmark Unidad 4 - El punto de vista del autor - Quizlet acumulativo

View Set

Biology Chapter4 Thinking Critically

View Set

ECON 103 Exam practice questions

View Set

ACG 4803 (CH 24 - Full Disclosure in Financial Reporting)

View Set

Chapter 7: Life Insurance Tax Considerations

View Set